Download as pdf or txt
Download as pdf or txt
You are on page 1of 42

UPPCS 2022 ANS- 6 (2056)

1. Answer. (c) ईत्तर. (c)


 In India, the Portuguese established their trading  भारत में कालीकट, गोवा, दमन, दीव एवं हुगली के
centers in the ports of Calicut, Goa, Daman, Diu बंदरगाहों में पुततगाललयों ने ऄपनी व्यापाररक कोरियों
की स्थापना की।
and Hooghly.
 भारत में लितीय पुततगाली ऄलभयान पेड्रो ऄल्वरे ज
 The second Portuguese expedition to India was कै ब्राल के नेतृत्व में छोडा गया। कै ब्राल ने कालीकट
launched under the leadership of Pedro Alvarez बंदरगाह से एक ऄरबी जहाज को पकडकर जमोररन
Cabral. Cabral captured an Arabic ship from the को ईपहार स्वरूप भेंट ककया।

port of Calicut and presented it to the Zamorin as


a gift.

2. Answer. (d) ईत्तर. (d)


Francisco de Almeida- फ्ांलसस्को द ऄल्मेडा-
 It came to India in 1505 as the first Viceroy of  यह पुततगाल के प्रथम वायसराय के रूप में 1505 इ. में
Portugal and ruled here till 1509. भारत अया और 1509 इ. तक यहााँ शासन ककया।
 It built such Portuguese fortifications in India,  आसने भारत में ऐसे पुततगाली दुगों का लनमातण ककया ,
whose purpose was not to protect but to establish लजनका ईद्देश्य सुरक्षा ना होकर हहद महासागर के
व्यापार पर पुततगाली लनयंत्रण स्थालपत करना था।
Portuguese control over the trade of the Indian
आसकी यह नीलत नीले ऄथवा शांत जल की नीलत
Ocean. Its policy was called the policy of blue or कहलाइ।
calm water.  यह 1508 इ. में संयुक्त मुलस्लम नौसैलनक बेडे (लमस्र+
 It was defeated in 1508 by a combined Muslim तुकी+ गुजरात) से चौल के युद्ध में परालजत हो गया।
naval fleet (Egypt + Turkey + Gujarat) in the  आसने सामुकिक नीलत को ऄलधक महत्व कदया।

Battle of Chaul.
 It gave more importance to maritime policy.

3.Answer. (d) ईत्तर. (d)


 The Portuguese established the first printing  पुततगाललयों ने सन 1556 इ. में गोवा में प्रथम हप्रटटग
press in Goa in 1556. प्रेस की स्थापना की।
 The Portuguese also brought crops such as  पुततगाली भारत में तंबाकू जैसी फसलें तथा अलू ,
tobacco to India, and fruits and vegetables such हभडी, लमचत, ऄनन्नास, सपाटा और मूंगफली जैसी फल
as potato, okra, chili, pineapple, and groundnut. और सलजजयां भी लाए।
 भारत में गोलथक वास्तुकला की शुरुअत पुततगाललयों
 The beginning of Gothic architecture in India is
के अगमन के साथ ही मानी जाती है।
considered to be with the arrival of the

©MakeIAS www.makeias.in 9899282107, 8700476287


Portuguese.

4. Answer. (d) ईत्तर. (d)


 The Dutch established their trading factories in  डचों ने गुजरात, बंगाल, लबहार, ईडीसा में ऄपनी
Gujarat, Bengal, Bihar, Orissa. व्यापाररक फै लरियों की स्थापना की।
 भारत में प्रथम डच फै रिी की स्थापना मसुलीपट्टनम में
 The first Dutch factory in India was established in
1605 इ. में हुइ। आसके पश्चात् 1610 इ. में पुललकट ,
Masulipatnam in 1605 AD. After Pulicat in 1610,
1616 इ. में सूरत में डच फै लरियों की स्थापना हुइ।
Dutch factories were established in Surat in 1616
डचों ने लचनसुरा और कोचीन में भी ऄपनी फै रिी
AD. The Dutch also established their factories at स्थालपत कीं।
Chinsurah and Cochin.  बंगाल में प्रथम डच फै रिी की स्थापना पीपली सन
 The first Dutch factory in Bengal was established 1627 इ. में की गइ।

at Peepli in 1627.

5. Answer. (c) ईत्तर. (c)


 The new sea route to India was discovered by  भारत के ललए नए समुिी मागत की खोज पुततगाली
the Portuguese merchant Vasco da Gama on 17 व्यापारी वास्कोलडगामा ने 17 मइ 1498 को भारत के
पलश्चमी तट पर लस्थत बंदरगाह कालीकट पहुंचकर
May 1498 by reaching the port of Calicut on the
की। वास्कोलडगामा का स्वागत कालीकट के तत्कालीन
west coast of India. Vasco da Gama was
शासक जमोररन (यह कालीकट के शासक की ईपालध
welcomed by the then ruler of Calicut, Zamorin थी) िारा ककया गया। तत्कालीन भारतीय व्यापार पर
(this was the title of the ruler of Calicut). This ऄलधकार रखने वाले ऄरब व्यापाररयों को जमोररन का
behavior of Zamorin was not liked by the Arab यह व्यवहार पसंद नहीं अया और ईन्होंने पुततगाललयों
का लवरोध ककया।
merchants who controlled the then Indian trade
and they opposed the Portuguese.

6. Answer. (b) ईत्तर. (b)


Alfonso de Albuquerque ऄल्फांसो द ऄलबुककत
 He came to India in 1509 as the second  यह लितीय पुततगाली वायसराय के रूप में 1509 इ. में
Portuguese Viceroy. भारत अया।
 आसे पुततगाली शलक्त का वास्तलवक संस्थापक माना
 It is considered the real founder of Portuguese
जाता है। आसने कोचीन को ऄपना मुख्यालय बनाया।
power. He made Cochin its headquarters.
 ऄल्बूककत ने 1510 इ. में गोवा को बीजापुर के शासक
 Albuquerque snatched Goa from Bijapur's ruler युसूफ अकदलशाह से छीनकर ऄपने ऄलधकार क्षेत्र में
Yusuf Adilshah in 1510 AD and put it under his कर ललया।
jurisdiction.  आसने पुततगाली पुरुषों को भारतीय लियों से लववाह
करने के ललए प्रोत्सालहत ककया।
 This encouraged Portuguese men to marry
 आसने गोवा को पुततगाली सत्ता एवं संस्कृ लत के
Indian women. महत्वपूणत कें ि के रूप में स्थालपत ककया।
 He established Goa as an important center of
Portuguese power and culture.

7. Answer. (b) ईत्तर. (b)

©MakeIAS www.makeias.in 9899282107, 8700476287


 The Portuguese used the Nagapattinam port  पुततगाली एलशयाइ देशों से व्यापार करने हेतु भारत में
ऄवलस्थत नागपट्टनम बंदरगाह का प्रयोग करते थे।
located in India to trade with Asian countries. The
पुततगाली चटगांव के बंदरगाह को महान बंदरगाह की
Portuguese used to call the port of Chittagong as संज्ञा देते थे।
the great port.  पुततगाललयों ने काटतज-अमेडा काकफला पद्धलत का
 The Portuguese used the Cartazes system, प्रयोग ककया, लजसके ऄंतगतत हहद महासागर का प्रयोग
under which every ship using the Indian Ocean करने वाले प्रत्येक जहाज को शुल्क ऄदा करना होता
था, लजन जहाजों को परलमट प्राप्त होता था ईन्हें
had to pay a fee, ships that had a permit were not
गोला-बारूद और काली लमचत का व्यापार करने की
allowed to trade in ammunition and pepper . ऄनुमलत नहीं होती थी।

8. Answer. (b) ईत्तर. (b)


 The Dutch were residents of Holland or the  डच होलेंड या नीदरलैंड के लनवासी थे। ईनकी आच्छा
दलक्षण पूवत एलशया के मसाला बाजारों में सीधा प्रवेश
Netherlands. Their desire was to establish control
करके लनयंत्रण स्थालपत करने की थी।
by direct entry into the spice markets of
 1596 इ. में कारनेललस दी हाईटमैन भारत अने वाला
Southeast Asia. प्रथम डच यात्री था।
 Cornelis de Houtman was the first Dutch traveler  डचों ने 1602 इ. में एक लवशाल व्यापाररक कं पनी की
to visit India in 1596 AD. स्थापना कक लजसे यूनाआटेड इस्ट आं लडया कं पनी ऑफ
 The Dutch established a huge trading company नेदरलैंड के नाम से जाना जाता था।

in 1602 , which was known as the United East


India Company of the Netherlands.

9. Answer. (c) ईत्तर. (c)


 Madras was founded in the year 1639 by Francis  मिास की स्थापना ककसने की सन् 1639 इ. में इस्ट
Day, an employee of the East India Company, by आं लडया कं पनी के कमतचारी फ़्ांलसस डे ने लवजयनगर के
राजा से कु छ भूलम लेकर की थी।
taking some land from the Raja of Vijayanagara.

10. Answer. (b) ईत्तर. (b)


 Jahangir gave permission to the British to  जहांगीर ने 1613 इ. में सूरत में ऄंग्रेजों को स्थायी
establish a permanent factory in Surat in 1613 कारखाना स्थालपत करने की ऄनुमलत प्रदान की।
AD.  जहांगीर के दरबार में 1615 इ. में सम्राट जेम्स प्रथम

 In the court of Jahangir in 1615 AD, the envoy of का दूत सर टॉमस रो पहुंचा, लजसने मुगल साम्राज्य के
सभी भागों में व्यापार करने एवं फै रिी स्थालपत करने
Emperor James I, Sir Thomas Roe arrived, who
का ऄलधकार प्राप्त ककया।
obtained the right to trade and set up factories in
all parts of the Mughal Empire.

11. Answer. (a) ईत्तर. (a)


 In 1661 AD, King Charles II of England was  1661 इ. में आं ग्लैंड के सम्राट चाल्सत लितीय का लववाह
married to the Portuguese Princess Catherine. पुततगाली राजकु मारी कै थरीन से हुअ।
 In this, marriage Bombay was presented to  आस लववाह में मुंबइ को दहेज के रूप में चाल्सत को भेंट
Charles as a dowry, which he gave to the कर कदया गया, लजसे ईन्होंने 10 पाईं ड वार्षषक ककराए

©MakeIAS www.makeias.in 9899282107, 8700476287


company at an annual rent of £10. पर कं पनी को दे कदया।

12.Answer. (d) ईत्तर. (d)


 In 1664 AD, during the time of Emperor Louis XIV  1664 इ. में फ्ांस के सम्राट लुइ 14वें के समय ईनके
of France, the efforts of his minister Colbert led to मंत्री कोल्बटत के प्रयासों से फ्ांसीसी व्यापाररक कं पनी
the establishment of the French trading company “कं पनी द आण्ड ओररएंटल” की स्थापना हुइ।
 फ्ैं क कै रो िारा फ्ांसीलसयों की प्रथम फै रिी की
"Company the Ind Oriental".
स्थापना सूरत में 1668 इ. में की गइ।
 The first factory of the French was established by
 फ्ांसीलसयों िारा दूसरी व्यापाररक कोिी की स्थापना
Franc Caron in Surat in 1668 AD. गोलकुं डा ररयासत के सुल्तान से ऄलधकार प्राप्त करने
 The second trading factory was established by के पश्चात् 1669 इ. में मसुलीपट्टनम में की गइ।
the French at Masulipatnam in 1669 AD after
obtaining authority from the Sultan of the princely
state of Golconda.

13.Answer. (a) ईत्तर. (a)


Nino de Cunha- नीनो डी कु न्हा-
 यह तीसरा पुततगाली गवनतर था।
 This was the third Portuguese governor.
 आसका प्रमुख कायत गोवा को पुततगाललयों की
 Its main function was to convert Goa into the
औपचाररक राजधानी के रूप में पररवर्षतत करना था।
former capital of the Portuguese.  आसने कोचीन से राजधानी पररवर्षतत करके गोवा को
 It changed the capital from Cochin to Goa. राजधानी बनाया।
 It established Portuguese settlements at Hooghly  आसने हुगली (बंगाल) और सेंथोमा (मिास) में

(Bengal) and St. Thomas (Madras). Bassein in पुततगाली बलस्तयों की स्थापना की। 1534 इ. में बेलसन
1534 AD and Diu in 1535 AD. तथा 1535 इ. में दीव पर ऄलधकार ककया।

14. Answer. (b) ईत्तर. (b)


 After the defeat of the British in the Battle of  1759 इ. में बेदरा के युद्ध में ऄंग्रेजों िारा हुइ पराजय
Bedra in 1759 AD, the Dutch finally collapsed in के ईपरांत भारत में ऄंलतम रूप से डचों का पतन हो
गया। आस युद्ध में ऄंग्रेजी सेना का नेतृत्व रलाआव िारा
India. The English army in this war was led by
ककया गया था।
Clive.

15. Answer. (b) ईत्तर. (b)


 The black hole tragedy is an incident in West  काल कोिरी नामक घटना पलश्चम बंगाल की एक
Bengal, which is from the pre-independence घटना है, जो स्वतंत्रता पूवत काल की है।

period.  ऐसा माना जाता है कक , बंगाल के नवाब

 It is believed that the Nawab of Bengal (Siraj-ud- (लसराजुद्दौला) ने 146 ऄंग्रेज़ बंकदयों , लजनमें लियााँ

Daulah) locked 146 British prisoners, including और बच्चे भी सलम्मललत थे , को एक 18 फु ट लंबे , 14

women and children, in an 18 feet long, 14 feet फु ट 10 आं च चौडे कमरे में बन्द कर कदया था।

10 inches wide room.  20 जून, 1756 इ. की रात को बंद करने के बाद , जब

 After closing it on the night of June 20, 1756, 23 जून को प्रातः कोिरी को खोला गया तो, ईसमें 23

©MakeIAS www.makeias.in 9899282107, 8700476287


when the cell was opened in the morning on June लोग ही जीलवत पाये गये।
23, only 23 people were found alive in it.  जीलवत रहने वालों में 'हालवैल' भी थे , लजन्हें ही आस
 Among those who survived was 'Holwell', who is घटना का रचलयता माना जाता है।
believed to be the author of this incident.  आस घटना की लवश्वसनीयता को आलतहासकारों ने
संकदग्ध माना है
 Historians have doubted the credibility of this
event.

16. Answer. (b) ईत्तर. (b)


Second Treaty of Allahabad (August 16, 1765 AD) आलाहाबाद की लितीय संलध (16 ऄगस्त, 1765 इ.)
 This treaty was concluded between Clive and  यह संलध रलाआव एवं शुजाईद्दौला के मध्य सम्पन्न
Shuja-ud-daula. हुइ।
आस संलध की शतें लनम्नवत थीं-
The terms of this treaty were as follows-
 आलाहाबाद और कडा को छोडकर ऄवध का शेष क्षेत्र
 The rest of Awadh, except Allahabad and Kara, नज्मुद्दौला को वापस कर कदया गया।
was returned to Najm-ud-daula.  कं पनी िारा ऄवध की सुरक्षा हेतु नवाब के खचत पर

 An English army was kept in Oudh by the एक ऄंग्रेजी सेना ऄवध में रखी गइ।
 कं पनी को ऄवध में कर मुफ्त व्यापार करने की सुलवधा
Company at the expense of the Nawab for the प्राप्त हो रही है
security of Awadh.  शुजाईद्दौला को बनारस के राजा बलवंत हसह से पहले
 The company is getting the facility of doing tax की ही तरह लगान वसूल करने का ऄलधकार कदया
गया। राजा बलवंत हसह ने युद्ध में ऄंग्रेजो की सहायता
free business in Awadh.
की थी।
 Shuja-ud-daula was given the right to collect rent
from the king of Banaras, Balwant Singh, as
before. Raja Balwant Singh helped the British in
the war.

17. Answer. (b) ईत्तर. (b)


First Treaty of Allahabad (August 12, 1765 AD) आलाहाबाद की प्रथम संलध (12 ऄगस्त, 1765 इ.)
 This treaty was concluded between Clive, Mughal यह संलध रलाआव , मुग़ल सम्राट शाहअलम लितीय तथा बंगाल
Emperor Shah Alam II and Najm-ud-daula, the के नवाब नज्मुद्दौला के मध्य सम्पन्न हुइ। आस संलध की शतो
लनम्नवत थी-
Nawab of Bengal. The terms of this treaty were
 कं पनी को मुग़ल सम्राट शाहअलम कितीय से बंगाल ,
as follows-
लबहार तथा ईडीसा की दीवानी प्राप्त हुइ।
 The Company received the Diwani of Bengal,  कं पनी ने ऄवध के नवाब से कडा और आलाहाबाद के
Bihar and Orissa from the Mughal Emperor Shah लजले लेकर मुग़ल , सम्राट शाहअलम कितीय को दे
Alam II. कदए।
 The Company took the districts of Kada and  कं पनी ने मुग़ल सम्राट को 26 लाख रूपये की वार्षषक

Allahabad from the Nawab of Awadh and gave it पेंशन देना स्वीकार ककया।

to the Mughal Emperor Shah Alam II.


 The company agreed to pay an annual pension
of 26 lakh rupees to the Mughal emperor.

©MakeIAS www.makeias.in 9899282107, 8700476287


18. Answer. (c) ईत्तर. (c)
 First Mysore War (1767 - 1769) - Lord Verelst  प्रथम मैसूर युद्ध(1767 - 1769)- लॉडत वेरेल्स्ट
 Second Mysore War (1780 - 1784) - Lord Warren  लितीय मैसूर युद्ध( 1780 - 1784)- लॉडत वारे न
Hastings हेहस्टग्स
 Third Mysore War (1790 - 1792) - Lord  तृतीय मैसूर युद्ध(1790 - 1792)- लॉडत कॉनतवॉललस

Cornwallis  चतुथत मैसूर युद्ध(1799)- लॉडत वेलेजली

 Fourth Mysore War(1799) - Lord Wellesley

19.Answer. (d) ईत्तर. (d)


 Tipu Sultan was a member of the Jacobin Club.  टीपू सुल्तान जैकोलबन रलब का सदस्य था।
 कहा जाता है कक टीपू सुल्तान ने ऄपनी राजधानी
 Tipu Sultan is said to have planted the tree of
श्रीरं गपट्टनम में स्वतंत्रता का वृक्ष लगवाया था।
freedom in his capital, Srirangapatnam.  टीपू सुल्तान ने नइ मुिा ढलवाइ तथा नाप-तोल की
 Tipu Sultan introduced new currency and नवीन पद्धलत ऄपनाइ।
adopted a new method of measurement.  ईसने अधुलनक कै लेंडर की शुरुअत की तथा श्रंगेरी के
मि को दान कदया।
 He introduced the modern calendar and donated  टीपू सुल्तान ने ऄपनी एक व्यापाररक कं पनी भी
it to the monastery of Sringeri. स्थालपत की थी।
 Tipu Sultan had also established his own trading
company.

20. Answer. (c) ईत्तर. (c)


Second Anglo-Maratha War लितीय अंग्ल-मरािा युद्ध
 लॉडत वेलेजली का ऐसा मानना था कक भारत को
 Lord Wellesley believed that the only way to
नेपोललयन के खतरे से बचने का एकमात्र ईपाय समस्त
avoid Napoleon's threat to India was to annex all भारतीय राज्यों का ऄलधग्रहण करना था। आस ईद्देश्य
the Indian states. To achieve this objective, its की प्रालप्त के ललए ईसके सहायक संलध प्रणाली का
subsidiary alliance system was developed. Nana लवकास ककया। नाना फडनवीस ऄंग्रेजों की कु रटलता से
पररलचत थे , आसललए ईन्होंने मरािों को सहायक संलध
Fadnavis was aware of the deviousness of the
से दूर रखा। सन 1801 में पेशवा बाजीराव लितीय ने
British, so he kept the Marathas away from the
जसवंत राव होल्कर के भाइ लबििू जी की हत्या कर दी
subsidiary alliance. In 1801, Peshwa Bajirao II तथा पूना पर अक्रमण कर पेशवा और हसलधया की
killed Jaswant Rao Holkar's brother Bithu ji and सेना को परालजत कर कदया। पूना पर होल्कर का
attacked Poona and defeated the army of लनयंत्रण हो गया। बाजीराव लितीय ने भागकर बेसीन
में शरण ली। 31 लसतम्बर, 1802 इ. को बाजीराव
Peshwa and Scindia. Holkar got control over
लितीय एवं ऄंग्रेजों के मध्य बेसीन की संलध हुइ।
Poona. Bajirao II fled and took refuge in the
basin. On September 31, 1802, the Treaty of
Bassein was signed between Bajirao II and the
British.

21. Answer. (b) ईत्तर. (b)


Treaty of Bassein: बेसीन की संलध:
 The company got Surat Nagar.  कं पनी को सूरत नगर लमल गया।

©MakeIAS www.makeias.in 9899282107, 8700476287


 The Peshwa accepted English protection.  पेशवा ने ऄंग्रेजी संरक्षण स्वीकार कर ललया।
 पेशवा ने पूना में ऄंग्रेजी सेना को रखना स्वीकार कर
 The Peshwa agreed to keep the English army in
ललया।
Poona.  पेशवा ने ऄंग्रेजो को लनजाम से चौथ वसूली का
 The Peshwa gave the British the right to collect ऄलधकार दे कदया तथा गायकवाड के लवरुद्ध युद्ध न
Chauth from the Nizam and promised not to fight करने का वचन कदया।
 पेशवा ने ऄपने लवदेशी मामले कं पनी के ऄधीन कर
against the Gaekwad. कदया।
 The Peshwa placed his foreign affairs under the  पेशवा ने कं पनी के ककसी भी यूरोपीय शत्रु को न
Company. रखना स्वीकार ककया।
 सन 1803 में भोले ने लववश होकर ऄंग्रेजों से देवगांव
 The Peshwa agreed not to keep any European
की संलध की आस संलध के ऄनुसार कटक एवं वधात नदी
enemy of the Company.
के पलश्चम का भाग ऄंग्रेजों को लमल गया
 In the year 1803, Bhole was compelled by this
treaty of the Treaty of Deogaon with the British,
according to this treaty, the British got the part of
Cuttack and the west of the Wardha river.

22. Answer. (b) ईत्तर. (b)


Reasons for the downfall of the Dutch डचों के पतन के कारण
 ऄंग्रेजों की तुलना में नौसैलनक शलक्त का कमजोर होना
 Weakness of naval power in comparison to the
 ऄत्यलधक कें िीकरण की नीलत ऄपनाना
British  अर्षथक लस्थलत में लगरावट अना
 adopting a policy of excessive centralization
 decline in economic condition

23. Answer. (a) ईत्तर. (a)


First Anglo-Sikh War (1845-46 AD) प्रथम अंग्ल-लसख युद्ध (1845-46 इ.)
 The first Anglo-Sikh war was the result of the  प्रथम अंगल-लसख युद्ध राजमाता लझन्दन की
राजनीलतक महत्वकांक्षाओ का पररणाम था। ऄंग्रेजी
political ambitions of Rajmata Jhindan. The
सेनानायक मेजर ब्रॉडफु ट की नीलतयों ने लसख
policies of the British general, Major Broadfoot, सेनापलत लालहसह तथा तेजाहसह को ऄंग्रजो के लवरुद्ध
provoked the Sikh generals Lal Singh and Teja संघषत करने के ललए ईकसाया। पररणामस्वरूप ,
Singh to fight against the British. As a result, the खालसा सरकार ने ऄंग्रजों के लवरुद्ध अक्रमण की
Khalsa government declared an attack against घोषणा कर दी।

the British.  आस प्रकार प्रथम अंग्ल-लसख युद्ध अरं भ हो गया। आस


युद्ध के समय भारत का गवनतर जनरल लॉडत हार्डडग था
 Thus began the First Anglo-Sikh War. At the time एवं भारत के प्रधान सेनापलत का नेतृत्व लॉडत गफ के
of this war, the Governor General of India was पास था।
Lord Hardinge and the Chief General of India  आस युद्ध में चार लडाआयााँ कफरोजशाह , मुदकी,
was headed by Lord Gough. बुद्धोवाल और ऄलीबाल में लडी गयी, लजनमे कोइ भी
 In this war, four battles were fought in Firoz लनणतय नहीं हो सका। पांचवीं एवं ऄंलतम लडाइ
सबराओ में हुइ जो लनणातयक लसद्ध हुइ। आस लडाइ में
Shah, Mudki, Budhewal and Alibal, in which no
ऄंग्रेजी सेना ने लसखों को परालजत कर कदया तथा 20
decision could be taken. The fifth and final battle
फरवरी, 1846 को लाहौर पर ऄलधकार कर ललया।

©MakeIAS www.makeias.in 9899282107, 8700476287


took place at Sabrao which proved to be आस युद्ध के पश्चात् 9 माचत, 1846 इ. को ऄंग्रेजों एवं
decisive. In this battle, the British army defeated लसखों के मध्य लाहौर की संलध हुइ।
the Sikhs and captured Lahore on February 20,
1846. After this war, on March 9, 1846, the
Treaty of Lahore was signed between the British
and the Sikhs.

24. Answer. (b) ईत्तर. ( b)


Second Anglo-Sikh War (1848 - 49 AD) लितीय अंग्ल-लसख युद्ध (1848 - 49 इ.)
 Three battles were fought in the Second Anglo-  लितीय अंग्ल लसख युद्ध में तीन लडाआयााँ लडी गयीं।
Sikh War. On November 22, 1848, the first battle 22 नवम्बर, 1848 इ. को सवतप्रथम रामनगर का युद्ध
लडा गया। आस युद्ध में ऄंग्रेजी सेना का नेतृत्व लॉडत गफ
of Ramnagar was fought. In this war, the English
तथा लसख सेना का नेतृत्व शेरहसह ने ककया।
army was led by Lord Gough and the Sikh army
 दूसरा, लचललयााँवाला का युद्ध था जो 13 जनवरी,
was led by Sher Singh.
1849 इ. को लडा गया। आस युद्ध में ऄंग्रेजी और लसख
 Second, there was the battle of Chillianwala
नेतृत्व में कोइ पररवततन नहीं अया। यह युद्ध भी
which was fought on January 13, 1849 AD. There
ऄलनणीत ही समाप्त हुअ।
was no change in the English and Sikh
 तदोपरांत, तृतीय गुजरात का युद्ध 13 फरवरी, 1849
leadership in this war. This war also ended को लडा गया। यह युद्ध लनणातयक लसद्ध हुअ। आसमें
inconclusive. ऄंग्रेजी सेना का नेतृत्व चाल्सत नेलपयर ने ककया। ऄंग्रेजो
 Thereafter, the Third War of Gujarat was fought ने लसख सेना को परालजत कर कदया। आसे 'तोपों के

on February 13, 1849. This battle proved युद्ध' के नाम से भी जाना जाता है।

decisive. In this, the English army was led by


Charles Napier. The British defeated the Sikh
army. It is also known as 'War of the Cannons'.

25. Answer. (d) ईत्तर. (d)


Lord Clive (1757-1760 and 1765-1767) - लाडत रलाआव (1757-1760 एवं 1765-1767) -
 Lord Clive was the first governor to be appointed  लाडत रलाआव इस्ट आं लडया कम्पनी िारा भारत में
लनयुक्त होने वाला प्रथम गवनतर था लजसे इस्ट आं लडया
by the East India Company in India, who was
कं पनी ने 1757 में बंगाल का गवनतर लनयुक्त ककया।
appointed governor of Bengal by the East India
 रलाआव ने बंगाल में िैध शासन की व्यवस्था की
Company in 1757.
 रलाआव के बाद, िैध शासन के दौरान वेरेल्स्ट (1767-
 Clive arranged diarchy in Bengal
1769) और कार्टटयर (1769-1772) बंगाल के गवनतर
 After Clive, Verelst (1767–1769) and Cartier रहे।
(1769–1772) were the governors of Bengal  1757 का प्लासी का युद्ध भी लाडत रलाआव के नेतृत्व
during the diarchy. में लडा गया।
 The Battle of Plassey of 1757 was also fought
under the leadership of Lord Clive.

26. Answer. (c) ईत्तर. (c)

©MakeIAS www.makeias.in 9899282107, 8700476287


Sir George Barlow (1805–1807) सर जॉजत बालो (1805-1807)
 Treaty of Rajpurghat of 1805 AD and Sepoy  1805 इ. की राजपुरघाट की संलध एंव 1806 इ. में
Mutiny of Vellore in 1806 AD वेल्लोर का लसपाही लविोह

27. Answer. (c) ईत्तर. (c)


Sir Charles Metcalfe (1835-1836) सर चाल्सत मैटकाफ (1835-1836)
 It ended the restrictions on newspapers in India,  आसने भारत में समाचार पत्रों पर लगे प्रलतबंधों को
hence it is called the liberator of the Indian press. समाप्त ककया , आस ललए आसे भारतीय प्रेस का
मुलक्तदाता कहा जाता है।

28. Answer. (b) ईत्तर. (b)


In 1939, Subhash Chandra Bose formed a new party  1939 इ. मे सुभाषचन्ि बोस ने 'णारवडत जलॉक' नाम
named 'Forward Bloc'. की एक नइ पाटी का गिन ककया।
 सुभाषचन्ि बोस के त्याग पत्र के पश्चात् डॉ. राजेन्ि
After the resignation of Subhash Chandra Bose, Dr.
प्रसाद को कांग्रेस का ऄध्यक्ष बनाया गया था।
Rajendra Prasad was made the President of Congress.

29. Answer. (a) ईत्तर. (a)


Lord Cornwallis (1786-1793) लाडत कानतवाललस (1786-1793)
 Lord Cornwallis is considered the father of the  लाडत कॉनतवॉललस को भारत में लसलवल सेवा एवं
पुललस व्यवस्था का जनक माना जाता है।
civil service and police system in India.
 तृतीय अंग्ल-मैसूर (1790-1792) युद्ध हुअ।
 Third Anglo-Mysore (1790-1792) war took place.
 1791 इ. को जोनाथन डंकन ने बनारस में संस्कृ त
 In 1791 AD, Jonathan Duncan established the
लवद्यालय की स्थापना की।
Sanskrit school in Banaras.
 कानतवललस कोड का लनमातण (1793) ककया।
 Cornwallis Code was created (1793).
 बंगाल में स्थायी बंदोबस्त प्रणाली (1793) लागू की
 The Permanent Settlement System (1793) was
 1805 में गाजीपुर में कानतवाललस की मृत्यु हो गयी।
introduced in Bengal.
 गाजीपुर में आसका मकबरा बनाया गया है , लजसे "लाट
 Cornwallis died in Ghazipur in 1805.
साहब' का मकबरा कहा जाता है।
 Its tomb has been built in Ghazipur, which is
called the tomb of "Lat Saheb".

30. Answer. (d) ईत्तर. (d)


 Second Anglo-Sikh War (1848-49 AD) and  लितीय अंग्ल-लसरख युद्ध (1848-49 इ.) तथा 1849
merger of Punjab with British rule in 1849 AD इ. में पंजाब का लब्ररटश शासन में लवलय
 Wood's Dispatch on Education (1854) came,  वुड का लशक्षा संबंधी लडस्पैच (1854) अया, लजसे
which is called 'Magna Carta' for education भारत में लशक्षा सुधारों के ललए ‘मैग्नाकाटात’ कहा जाता
reforms in India. है|
 In 1852 “Inam Commission” was established,  1852 में “आनाम कमीशन” की स्थापना की गयी थी ,
whose objective was to take the land tax-free लजसका ईद्देश्य भूलमकर रलहत जागीरों को ऄंग्रेजी
शासन के ऄधीन ले लेना था।
Jagirs under British rule.
 पेशवा के पेंशन की समालप्त (1853)।
Abolition of Peshwa's pension (1853).

©MakeIAS www.makeias.in 9899282107, 8700476287


31. Answer. (c) ईत्तर. (c)
 In 1632, the British got a golden farman from the  1632 इ. में ऄंग्रेज़ों को गोलकुं डा के सुल्तान से सुनहरा
Sultan of Golconda. Under this, the company got फरमान लमला। आसके तहत कं पनी को ऄलधकार लमला
the right to trade freely with all the ports of कक वे 500 पैगोडा वार्षषक कर के बदले गोलकुं डा के
सभी बन्दरगाहों से स्वतंत्रतापूवतक व्यापार कर सकते
Golconda in return for an annual tax of 500
थे।
pagodas.
 1651 इ. में शाहशुजा ने 3000 वार्षषक कर के बदले
 In 1651, Shah Shuja allowed free trade बंगाल में चुंगी मुक्त व्यापार करने की स्वतंत्रता दे दी।
allowance in Bengal in exchange for 3000 annual
tax.

32. Answer. (a) ईत्तर. (a)


 The founder of Calcutta was Job Charnock.  कलकत्ता का संस्थापक जॉब चॉरनाक था।
 बंबइ का वास्तलवक संस्थापक जेराल्ड ऄंलगयार था।।
 The real founder of Bombay was Gerald Aungier.
 मिास का संस्थापक फ्ांलसस डे था , जबकक फ्ें को
 The founder of Madras was Francis Day, while
मार्टटन पांलडचेरी का प्रथम फ्ााँसीसी गवनतर था।
Franco Martin was the first French governor of
Pondicherry.

33. Answer. (a) ईत्तर. (a)


 Awadh - Saadat Khan Burhanul Mulk  ऄवध - सअदत खााँ बुरहानुलमुल्क
 हैदराबाद - लचनककललच खााँ लनजामुलमुल्क
 Hyderabad - Chinkilich Khan Nizamulmulk
 रुहेलखंड - वीर दाउद एवं ऄली मुहम्मद खााँ
 Rohilkhand - Veer Dawood and Ali Muhammad  बंगश राज्य - मुहम्मद खााँ बंगश
Khan  बंगाल - मुर्षशद कु ली खााँ
 Kingdom of Bangash - Muhammad Khan  कनातटक - सादुल्ला खााँ

Bangash
 Bengal - Murshid Quli Khan
 Karnataka - Sadulla Khan

34. Answer. (d) ईत्तर. (d)


 In addition to the above exemptions in the decree  फरुत खलसयर िारा 1717 इ. में जारी फरमान में
issued by Farrukhsiyar in 1717, there was also ईपरोक्त छू टों के ऄलतररक्त एक छू ट यह भी थी कक
कं पनी के बंबइ टकसाल में ढाले गए लसक्कों का समस्त
an exemption that the coins minted in the
भारत में प्रचलन होगा। ऄतः कथन 4 गलत हैं। ऄन्य
Bombay Mint of the Company would be in
तीनों कथन सही है।
circulation all over India. Hence statement 4 is
wrong. All other three statements are correct.

35. Answer. (d) ईत्तर. (d)


 In Bombay on 28 December 1885, The Indian  28 कदसम्बर 1885 इ. को बम्बइ में ए. ओ. ह्यूम के
National Congress was established under the नेतृत्व में भारतीय राष्ट्रीय कांग्रेस की स्थापनाहुइ।
 भारत एंव ऄफगालनस्तान के मध्य सीमा रे खा हेतु
leadership of A.O. Hume.
डू रण्ड अयोग (1893) की स्थापना।

©MakeIAS www.makeias.in 9899282107, 8700476287


 Establishment of Durand Commission (1893) for  वनातरयूलर प्रेस एरट (1878) पाररत ककया गया।
the boundary line between India and Afghanistan.  हप्रस ऑण वेल्स एडवडत तृतीय की भारत यात्रा
 The Vernacular Press Act (1878) was passed. (1875)।

 Visit of Prince of Wales Edward III to India


(1875).

36. Answer. (d) ईत्तर. (d)


 The Anglo-French war in India is known as the  भारत में हुए ऄंग्रेज़ फ्ााँसीसी युद्ध को ‘कनातटक युद्ध ’ के
'Carnatic War'. After the defeat in this war, the नाम से जाना जाता है। आस युद्ध में पराजय के पश्चात्
फ्ााँसीलसयों का ऄलधकार लसफत पांलडचेरी तक सीलमत
authority of the French remained limited to
रह गया।
Pondicherry only.

37. Answer. (a) ईत्तर. (a)


Lord Lytton (1876-1880) लाडत ललटन (1876-1880)
 He also had a nickname 'Owen Meredith'.  आनका एक ईपनाम 'ओवेन मेरेलडथ' भी था।

38. Answer. (b) ईत्तर. (b)


 Bahadur Shah I - Shah-e- Bekhabar  बहादुरशाह प्रथम - शाहे बेख़बर
 जहााँदारशाह - लम्पट मूखत
 Jahandar Shah - Lustful fool
 णरूतख़लसयर - घृलणत कायर
 Farrukhsiyar - Disgusting coward  मुहम्मदशाह - रं गीला
 Muhammad Shah - Rangeela

39. Answer. (b) ईत्तर. (b)


 The title of 'Maharaja' was given to Ajit Singh of  मारवाड के ऄजीत हसह को 'महाराजा' की पदवी दी
Marwar and appointed him the ruler of Gujarat. और गुजरात का शासक लनयुक्त ककया। ईसने जलजया
He also abolished the jizya tax. कर को भी समाप्त कर कदया।
 ज़़ुलल्णकार ख़ााँ ने साम्राज्य की लवत्तीय लस्थलत को
 Zulfiqar Khan tried to improve the financial
सुधारने का प्रयास ककया , ककन्तु ईसने 'आजारा
condition of the empire, but he promoted the
व्यवस्था' को बढावा कदया। आसके ऄन्तगतत एक लनलश्चत
'Izara system'. Under this, in return for collecting
दर पर भू-राजस्व वसूल करने के बदले में सरकार ने
land revenue at a fixed rate, the government had
'आजारे दार' (लगान के िे केदारों) और लबचैललयों के
started making agreements with 'inzadars' साथ करार करना अरम्भ कर कदया था कक वे सरकार
(revenue contractors) and middlemen to pay the को एक लनलश्चत मुिा रालश दें। बदले में ककसानों से
government a fixed amount of money. In return, लजतना लगान वसूल कर सकें , ईतना वसूलने के ललए

they were left free to collect as much rent from ईन्हें अज़ाद छोड कदया गया। आससे ककसानों का
ईत्पीडन बढा।
the peasants as they could. This increased the
oppression of the farmers.

40. Answer. (b) ईत्तर. (b)


 In 1904, the 'Ancient Monuments Protection Act'  1904 में 'प्राचीन स्मारक संरक्षण ऄलधयम ' पाररत
was passed. ककया।

©MakeIAS www.makeias.in 9899282107, 8700476287


 In 1912, a bomb was thrown at Lord Hardinge II,  1912 में लॉडत हार्डडग लितीय पर एक बम फें का गया,
an incident that became famous as the Delhi यह घटना कदल्ली बम काण्ड के नाम से प्रलसद्ध हुइ।
Bombing incident.  के रल में मोपला लविोह (1921)

 Moplah Rebellion in Kerala (1921)  1882 में स्थानीय शासन प्रणाली की शुरुअत हुइ।

 Local government system was introduced in


1882.

41. Answer. (a) ईत्तर. (a)


 Curzon also constituted an 'Irrigation  कज़तन ने 1901 इ. में 'सर कॉललन स्कॉट मॉनक्रीण' की
Commission' under the chairmanship of 'Sir Colin ऄध्यक्षता में एक 'हसचाइ अयोग' का भी गिन ककया।
Scott Moncrieff' in 1901.

42. Answer. (a) ईत्तर. (a)


 Alamgir II (1699-1759) was the 16th Mughal  अलमगीर लितीय ( 1699- 1759)16वााँ मुग़ल
emperor, who ruled from 1754 to 1759 AD. बादशाह था , लजसने 1754 से 1759 इ. तक राज्य
 Alamgir II was the son of Jahandar Shah, the ककया।
eighth Mughal emperor.  अलमगीर लितीय अिवें मुग़ल बादशाह जहााँदारशाह
 He was a very weak man, and he was a puppet का पुत्र था।

in the hands of his vizier Ghaziuddin Imad -Ul-  वह बडा कमज़ोर व्यलक्त था , और वह ऄपने वज़ीर

Mulk. ग़ाज़ीईद्दीन आमादुलमुल्क के हाथों की किपुतली था।

 Alamgir II is also known as 'Azizuddin'.  अलमगीर लितीय को 'ऄजीजुद्दीन' के नाम से भी

 Vazir Ghaziuddin made Alamgir II the emperor in जाना जाता है।

1754 AD. He wanted the emperor to remain a  वज़ीर ग़ाज़ीईद्दीन ने 1754 इ. में अलमगीर लितीय
को बादशाह बनाया। वह चाहता था कक बादशाह
puppet in his hand. This was a time of great
ईसके हाथ की किपुतली बना रहे। यह समय बडी
upheaval. ईथल-पुथल का समय था।
 Only after this the Marathas marched on Delhi in  आसके बाद ही मरािों ने 1758 इ. में कदल्ली पर चढाइ
1758 AD and conquered Punjab and drove Timur की और पंजाब को जीतकर तैमूर को वहााँ से लनकाल
out of there. कदया।

43. Answer. (a) ईत्तर. (a)


 Shah Alam II (reigned 1759-1806 AD) was the  शाहअलम लितीय (शासन काल 1759-1806 इ)
17th Mughal emperor. Its real name was 17वााँ मुग़ल बादशाह था। आसका ऄसली नाम
Shahzada Ali Gauhar. He ascended the throne in शाहज़ादा ऄली गौहर था। यह अलमगीर लितीय के
1759 AD as the successor of Alamgir II. ईत्तरालधकारी के रूप में 1759 इ. में गद्दी पर बैिा।
 बादशाह शाहअलम लितीय ने इस्ट आं लडया कम्पनी से
 Emperor Shah Alam II had made a treaty of
आलाहाबाद की सलन्ध कर ली थी और वह इस्ट आं लडया
Allahabad with the East India Company and was कम्पनी की पेंशन पर जीवन-यापन कर रहा था।
living on the pension of the East India Company.  1759 इ. में राजगद्दी पर बैिने के साथ ही ऄली गौहर
 In 1759 AD, as soon as he ascended the throne, ने बादशाह होने पर 'अलमशाह लितीय ' का लख़ताब
Ali Gauhar assumed the title of 'Alamshah II' धारण ककया। आलतहास में वह 'शाहअलम लितीय के

©MakeIAS www.makeias.in 9899282107, 8700476287


when he was the emperor. In history, he is नाम से प्रलसद्ध है।'
famous as 'Shah Alam II'.  आसके काल में ईत्तर-पलश्चम में ऄहमदशाह ऄजदाली ने
 During this period Ahmad Shah Abdali started his ऄपने हमले शुरू कर कदये थे। 1756 इ. में ईसने

attacks in the north-west. In 1756 AD he कदल्ली को लूटा और 1759 इ. में मरािों को , लजन्होंने

plundered Delhi and in 1759 AD the Marathas, 1758 इ. में पंजाब पर ऄलधकार कर ललया था , वहााँ

who had captured Punjab in 1758 AD, were से लनकाल बाहर ककया।

driven out from there.

44. Answer. (c) ईत्तर. (c)


War of Plassey प्लासी का युद्ध
On 13 June 1757, Clive started his military campaign  13 जून 1757 को रलाआव ने ऄपने सैन्य ऄलभयान की
from Chandranagar and soon reached Plassey. The शुरुअत चंिनगर से की और शीघ्र ही प्लासी पहुंच
गया। नवाब की सेना का नेतृत्व राजिोही मीरजाफर
Nawab's army was led by the traitor Mir Jafar.
कर रहा था।
 In this war, the reliable soldiers of the Nawab, Mir
 आस युद्ध में नवाब के लवश्वसनीय सैलनक मीरमदान एवं
Madan and Mohanlal, got martyrdom.
मोहनलाल को वीरगलत प्राप्त हुइ।
 Siraj-ud-Daulah had to flee to Murshidabad,  लसराजुद्दौला को भागकर मुर्षशदाबाद में शरण लेनी
where he was murdered by Mir Jafar's son, पडी, जहां मीरजाफर के पुत्र मीरन ने ईसकी हत्या
Meeran. This war took place on 30 June 1757. कर दी। यह युद्ध 30 जून 1757 को हुअ था।

45. Answer. (b) ईत्तर. (b)


 The Battle of Buxar was fought between Nawab  बरसर का युद्ध बंगाल के नवाब मीरकालसम , ऄवध के
Mir Qasim of Bengal, Nawab Shuja-ud-daula of नवाब शुजाईद्दौला एवं मुगल शासक शाह अलम
लितीय की संयुक्त सेना और ऄंग्रेजों के मध्य लडा गया
Awadh and the combined forces of Mughal ruler
था। आस युद्ध में ऄंग्रेजों की लवजय के पश्चात् बंगाल में
Shah Alam II and the British. After the victory of ऄंग्रेजी सत्ता का प्रभुत्व स्थालपत हो गया।
the British in this war, the dominance of English  1765 में रलाआव बंगाल के गवनतर के रूप में दूसरी
power in Bengal was established. बार कोलकाता अया। बरसर के युद्ध की समालप्त के
 In 1765, Clive came to Kolkata for the second बाद रलाआव ने मुगल सम्राट शाह अलम लितीय तथा
ऄवध के नवाब शुजाईद्दौला के साथ क्रमशः
time as the Governor of Bengal. After the end of आलाहाबाद की प्रथम एवं लितीय संलध की।
the Battle of Buxar, Clive made the first and
second treaty of Allahabad with Mughal Emperor
Shah Alam II and Nawab Shuja-ud-daula of
Awadh respectively.

46. Answer. (a) ईत्तर. (a)


 Champaran Satyagraha (1917)  चम्पारण सत्याग्रह (1917)
 Kheda Rebellion (1918).  खेडा लविोह (1918)।
 Third Anglo-Afghan War (1919)  तृतीय अंग्ल-ऄणग़ान युद्ध (1919)
 'Khilafat Movement' (1920-21).  'लख़लाणत अंदोलन' (1920-21)।

©MakeIAS www.makeias.in 9899282107, 8700476287


47. Answer. (c) ईत्तर. (c)
 Akbar II was the 18th emperor of the Mughal  ऄकबर लितीय मुग़ल वंश का 18वााँ बादशाह था। वह
dynasty. He was the son of Shah Alam II and शाहअलम लितीय का पुत्र था और ईसने 1806-
ruled from 1806-1837 AD. By that time, most of 1837 इ. तक राज ककया। ईसके समय तक भारत का
the Indian state had gone into the hands of the ऄलधकांश राज्य ऄंग्रेज़ों के हाथों में चला गया था और
British and in 1803, Delhi was also captured by 1803 इ. में कदल्ली पर भी ईनका कजज़ा हो गया।

them.  बादशाह शाहअलम लितीय ( 1769-1806 इ.) ऄपने


जीवन के ऄलन्तम कदनों में इस्ट आं लडया कम्पनी की
 Emperor Shah Alam II (1769-1806 AD) lived on
पेंशन पर जीवन यापन करता था।
the pension of the East India Company in the last  ईसका पुत्र बादशाह ऄकबर लितीय इस्ट आं लडया
days of his life. कम्पनी की कृ पा के सहारे नाम मात्र का ही बादशाह
 His son, Emperor Akbar II, was a nominal था।
 ऄकबर लितीय से गवनतर-जनरल लॉडत हेहस्टग्स
emperor with the help of the East India
(1813-1823) की ओर से कहा गया कक वह कम्पनी
Company's grace. के क्षेत्र पर ऄपनी बादशाहत का दावा छोड दे।
 Akbar II was asked by the Governor-General
Lord Hastings (1813–1823) to give up his
sovereign claim over the Company's territory.

48. Answer. (d) ईत्तर. (d)


Aurangzeb's policy of religious intolerance  औरं गजेब की धार्षमक ऄसलहष्णुता की नीलत
 लवदेशी अक्रमण
 Alien invasion
 सैन्य दुबतलता
 Military weakness  ऄयोग्य ईत्तरालधकारी
 Disqualified successor

49. Answer- (a) ईत्तर- (a)


Alfonso de Albuquerque ऄल्फांसो द ऄलबुककत
 It came to India in 1509 AD as the second  यह लितीय पुततगाली वायसराय के रूप में 1509 इ. में
Portuguese Viceroy. भारत अया।
 आसे पुततगाली शलक्त का वास्तलवक संस्थापक माना
 It is considered the real founder of Portuguese
जाता है। आसने कोचीन को ऄपना मुख्यालय बनाया।
power. It made Cochin its headquarters.
 ऄल्बूककत ने 1510 इ. में गोवा को बीजापुर के शासक
 Albuquerque snatched Goa from Bijapur's ruler युसूफ अकदलशाह से छीनकर ऄपने ऄलधकार क्षेत्र में
Yusuf Adilshah in 1510 AD and put it under his कर ललया।
jurisdiction.  आसने पुततगाली पुरुषों को भारतीय लियों से लववाह
करने के ललए प्रोत्सालहत ककया।
 This encouraged Portuguese men to marry
Indian women.

50. Answer- (a) ईत्तर- (a)


 In 1611, a trading factory was established by the  1611 इ. में ऄंग्रेजों िारा व्यापाररक कोिी की
British in Masulipatnam. स्थापना मसूलीपट्टनम में की गइ।
 पूवी भारत में ऄंग्रेजों िारा स्थालपत प्रथम कारखाना
 The first factory established by the British in

©MakeIAS www.makeias.in 9899282107, 8700476287


Eastern India was established in 1633 at 1633 इ. में ईडीसा के बालासोर में स्थालपत ककया
Balasore in Orissa. गया।

51. Answer. (b) ईत्तर. (b)


The French Governor Dupleix was the first to start giving  फ्ांसीसी गवनतर डू प्ले ने सवतप्रथम सहायक संलध
प्रणाली का प्रयोग करते हुए भारतीय राजाओं को
aid to the Indian kings and taking money from them in
सहायता देने व बदले में ईनसे धन लेने की शुरुअत की
return using the subsidiary alliance system. But the थी। परन्तु सहायक संलध की व्यापकता वेलेजली के
prevalence of subsidiary alliance was seen in the time of काल में देखने को लमली , जब ईसने आसे ऄंग्रेजी राज्य
Wellesley, when he made it a means of expansion of the के लवस्तार का साधन बनाया।
English kingdom.

52.ईत्तर. (a) ईत्तर. (a)


States acquired under doctrine of Lapse व्यपगत लसद्धांत के तहत ऄलधगृहीत राज्य

States year राज्य लतलथ

Satara 1848 सतारा 1848

Jaitpur 1849 जैतपुर 1849

Sambalpur 1849 संभलपुर 1849

Bhagat 1850 बघाट 1850

Udaipur 1852 ईदयपुर 1852

Jhansi 1852 झांसी 1852

Nagpur 1854 नागपुर 1854

53. Answer. (d) ईत्तर. (d)


British policy towards the princely states देशी ररयासतों के प्रलत लब्ररटश नीलत
 Although the British rule did not have any fixed यद्यलप लब्ररटश शासन की ररयासतों के प्रलत कोइ लनलश्चत नीलत
policy towards the princely states, yet they नहीं थी , तथालप ईन्होंने देश , कारक व पररलस्थलत के ऄनुसार
राजनैलतक सवोच्चता को प्राप्त करने के ललए लवलभन्न प्रकार की
followed different types of policies to achieve
नीलतयों का ऄनुसरण ककया। लवललयम ली वानतर ने ऄपनी
political supremacy according to the country,
पुस्तक ' द नेरटव स्टेटस ऑण आं लडया ' में ररयासतों के प्रलत
factor and situation. William Lee Warner in his लब्ररटश नीलत को तीन चरणों में वगीकृ त ककया है-
book 'The Native Status of India' classified the (क) घेरे की नीलत, 1765-1813 इ.
British policy towards the princely states into (ख) ऄधीनस्थ पृथक्करण की नीलत, 1813-1858 इ.
three phases- (ग) ऄधीनस्थ संघ की नीलत, 1858-1935 इ.
(a) The policy of siege, 1765-1813 AD.
(b) Policy of Subordinate Separation, 1813-1858 AD.

©MakeIAS www.makeias.in 9899282107, 8700476287


(C) Policy of Subordinate Union, 1858-1935 AD.

54. Answer. (b) ईत्तर. (b)


 Cornwallis made 'Sunset clause' in 1794. Under  कानतवाललस ने 1794 इ. में 'सूयातस्त कानून ' बनाया।
this it was declared that the Zamindari would be आसके तहत यह घोलषत ककया गया कक यकद पूवत
लनधातररत लतलथ के सूयातस्त तक सरकार को लगान नहीं
auctioned if the rent was not paid to the
कदया गया तो जमींदारी नीलाम हो जाएगी।
government by the sunset of the predetermined
date.

55. Answer. (d) ईत्तर. (d)


Revolt of poligars पोललगारों का लविोह
 पोललगार दलक्षण भारत में जमींदारों का एक वगत था
 The Poligars were a class of landlords in South
जो लोगो से कर की वसूली भी करता था। आस हबदु पर
India who also collected taxes from the people. ईनके और कं पनी के लहतों में टकराव ईत्पन्न हुअ
At this point a conflict of interest arose between रयोंकक कं पनी के राजस्व की वृलद्ध के ललए पोललगारों
him and the Company as control over the पर लनयंत्रण अवश्यक था।
 पोलीगार लविोह को दलक्षण भारतीय लविोह की संज्ञा
poligars was necessary for the growth of the
भी दी जाती है। लविोह के दमन के पश्चात् 1801 में
Company's revenue. कनातटक की संलध पर हस्ताक्षर ककये गए लजससे ऄंग्रेजों
 The Poligar Rebellion is also known as the South ने वत्ततमान तलमलनाडु के ऄलधकांश लहस्सों पर सीधा
Indian Rebellion. After the suppression of the लनयंत्रण स्थालपत कर ललया।

revolt, the Treaty of Carnatic was signed in 1801,


by which the British established direct control
over most of present-day Tamil Nadu.

56. Answer. (b) ईत्तर. (b)


Velu Thampi Rebellion वेलु थम्पी लविोह
 In 1805, the ruler of Travancore was removed for  1805 में वेलेजली की सहायक संलध को स्वीकार न
not accepting the Subsidiary Treaty of Wellesley. करने के कारण त्रावणकोर के शासक को हटा कदया
As a result, Velu Thampi, the Diwan of गया। पररणामस्वरूप 1808-09 में त्रावणकोर के
दीवान वेलु थम्पी ने लब्ररटश सरकार के लवरुद्ध लविोह
Travancore, revolted against the British
कर कदया
government in 1808-09.
 आस लविोह को ‘1857 के लविोह का पूवतगामी’भी कहा
 This rebellion is also called the 'Precursor of the जाता है।
Revolt of 1857'.

57.Answer. (a) ईत्तर. (a)


Paika Rebellion (1817-25) पाआका लविोह (1817-25)
 Due to the overthrow of the Raja of Khurda by  कं पनी िारा खुदात के राजा को ऄपदस्थ करने के कारण
क्षेत्र का प्रशासन कं पनी के ऄंतगतत अ गया लजस कारण
the Company, the administration of the area
पाआकों सैलनकों को पदच्युत कर कदया गया।
came under the Company, due to which the  कर तथा भू-राजस्व की दर में ऄत्यलधक वृलद्ध के कारण
Paikon soldiers were deposed. कृ षकों पर भी लवपरीत प्रभाव पडा। आन कारकों ने

©MakeIAS www.makeias.in 9899282107, 8700476287


 Due to excessive increase in the rate of tax and बेरोजगार हो गये कृ षकों तथा सैलनकों को साथ ला
कदया।
land revenue, the farmers were also adversely
 पाआक ईडीसा के पारं पररक सैलनकों को कहा जाता था
affected. These factors brought together the जो युद्ध काल में सैलनक तथा शांलतकाल में पुललस का
unemployed farmers and soldiers. कायत करते थे।
 Paiks were the traditional soldiers of Orissa who
served as soldiers in wartime and police in
peacetime.

58. Answer. (c) ईत्तर. (c)


Gadkari Rebellion गडकरी लविोह
 Babaji Ahirekar was the main leader of this  बाबाजी ऄलहरे कर आस लविोह के प्रमुख नेता थे।
rebellion.  1844 में ये लविोह गडकररयों िारा ककया गया जो
 The revolt in 1844 was carried out by the मरािा क्षेत्र के दुगों में सैलनकों के रूप में काम ककया
करते थे।
Gadkaris who worked as soldiers in the
fortifications of the Maratha region.

59. Answer. (c) ईत्तर. (c)


 The Pagalpanthi rebellion was started by 'Tipu'. पागलपंथी लविोह
 Pagalpanthis were a type of semi-religious sect.  पागलपंथी लविोह की शुरुअत 'टीपू' ने की थी।
Which was run by 'Karamshah' in North Bengal.  पागलपंथी एक प्रकार का ऄद्धत-धार्षमक सम्प्रदाय था।
 In 1825, the successor son of Karamshah, Tipu, जो ईत्तरी बंगाल में 'करमशाह' िारा चलाया गया था।

revolted against the tyranny of the zamindars.  1825 इ. मे करमशाह के ईत्तरालधकारी पुत्र 'टीपू' ने
ज़मींदारों के ऄत्याचार के लख़लाण लविोह ककया।
 This rebellion continued in this region from 1840
 यह लविोह आस क्षेत्र में 1840 इ. से 1850 इ. तक
to 1850.
जारी रहा।

60. Answer. (a) ईत्तर. (a)


 During the Revolt of 1857, the commanding  1857 के लविोह के दौरान बैरकपुर में कमांहडग
officer at Barrackpore was Hearsey. ऑकफसर हैरसे था।

61. Answer. (a) ईत्तर. (a)


Kuka Rebellion कू का लविोह
 The Kuka Rebellion started in Punjab in 1860-  कू का लविोह की शुरुअत पंजाब में 1860-1870 इ. में
हुइ थी।
1870 AD.
 पलश्चमी पंजाब में 'कू का लविोह ' की शुरुअत लगभग
 The 'Kuka Rebellion' in western Punjab was
1840 इ. में 'भगत जवाहर मल' िारा की गयी थी।
started by 'Bhagat JawaharMal' in about 1840.
 भगत जवाहर मल को 'लसयान साहब ' के नाम से भी
 Bhagat Jawahar Mal was also known as 'Sian
जाना जाता था।
Sahib'.
 लसयान साहब ने ऄपने लशष्य 'बालक हसह ' के साथ
 Sian Saheb along with his disciple 'Balak Singh' लमलकर ऄपने ऄनुयालययों का एक दल गरित
formed a group of his followers, whose ककया,लजसका मुख्यालय 'हजारा' में था।

©MakeIAS www.makeias.in 9899282107, 8700476287


headquarters was at 'Hazara'.  आस लविोह के लवरुद्ध ऄपनी दमनकाररयों नीलतयों को
 Adopting their repressive policies against this ऄपनाते हुये ऄंग्रेज़ों ने 1872 इ. में आसके एक नेता

rebellion, the British exiled one of its leaders 'रामहसह कू का ' को रं गून लनवातलसत कर कदया और
अन्दोलन पर लनयन्त्रण पा ललया गया।
'Ram Singh Kuka' to Rangoon in 1872 and the
movement was controlled.

62. Answer. (b) ईत्तर. (b)


Santhal Rebellion संथाल लविोह
 The Santhal rebellion was against the atrocities  संथाल लविोह लब्ररटश शासनकाल में ज़मींदारों तथा
साहूकारों िारा ककये जाने वाले ऄत्याचारों के लख़लाण
committed by the zamindars and moneylenders
ककया गया था। अकदवासी लविोहों में संथालों का यह
during British rule. This was the strongest सबसे सशक्त लविोह था।
rebellion of the Santhals among the tribal revolts.  भागलपुर से राजमहल के बीच का संथाल बहुल क्षेत्र
 The Santhal dominated area between Bhagalpur 'दामन-ए-कोह' के नाम से जाना जाता था।

to Rajmahal was known as 'Daman-i-Koh'.  संथालों ने गैर-अकदवालसयों को भगाने और ईनकी


सत्ता समाप्त कर ऄपनी सत्ता स्थालपत करने के ललए
 The Santhals waged a vigorous struggle to expel जोरदार संघषत छे डा।
the non-tribals and establish their power by  लविोह के नेतृत्वकतात 'लसद्धू' तथा 'कान्हू' थे ।
ending their power.  संथाल लविोह अर्षथक कारणों से लबहार और ईडीसा
 The leaders of the rebellion were 'Sidhu' and के वीरभूम , हसहभूम, बांकुडा, मुंगेर, हज़ारीबाग़ और
'Kanhu'. भागलपुर के लज़ले में हुअ था।
 ऄंग्रेज़ सरकार ने एक बडी सैन्य कायतवाही के बाद
 The Santhal rebellion took place in the districts of
1856 इ. में आस लविोह को दबाने में सफलता पाइ।
Birbhum, Singhbhum, Bankura, Munger,
सरकार ने ऄलग संथाल परगना की मांग को मानकर
Hazaribagh and Bhagalpur of Bihar and Orissa क्षेत्र में शांलत स्थालपत की।
due to economic reasons.
 The British government succeeded in
suppressing this rebellion in 1856 AD after a
major military operation. The government
accepted the demand for a separate Santhal
Pargana and established peace in the region.

63. Answer. (a) ईत्तर. (a)


Khond Rebellion खोंड लविोह
 Attempts by the government to ban human  सरकार िारा मानव बलल (मोररया प्रथा) को
sacrifice (Moriya system), imposition of new प्रलतबलन्धत करने का प्रयास ,नये कर अरोलपत करना
एवं ईनके क्षेत्रों में ज़मींदारी और साहूकारों का प्रवेश।
taxes and entry of zamindari and moneylenders
 खोण्ड लविोह की शुरुअत 1837 इ. में हुइ थी।
in their areas.
 ईडीसा के सबसे बडे जनजातीय समूह खोण्डों का
 The Khond rebellion started in 1837 AD. लवस्तार तलमलनाडु से लेकर बंगाल एवं मध्य भारत के
 The largest tribal group of Orissa, the Khonds पहाडी क्षेत्रों तक था।आनका स्थानीय नाम कु इ है।
extended from Tamil Nadu to Bengal and the hilly  आस लविोह का नेतृत्व 'चक्र लबसोइ ' नामक व्यलक्त ने
ककया था।

©MakeIAS www.makeias.in 9899282107, 8700476287


regions of central India. Their local name is Kui.
 This rebellion was led by a person named
'Chakra Bisoi'.

64. Answer. (b) ईत्तर. (b)


Bhil rebellion भील लविोह
 These rebellions took place in the region of यह लविोह महाराष्ट्र तथा राजस्थान के क्षेत्र में हुए थे।
महाराष्ट्र का भील लविोह
Maharashtra and Rajasthan.
 भील लविोह 1812 इ. में ऄंग्रेज़ों के लवरुद्ध ककया गया
Bhil Rebellion oMaharashtra
था।
 The Bhil rebellion was done against the British in  भील जालत के लोग पलश्चमी तट पर लस्थत ख़ानदेश में
1812. लनवास करते थे। आन लोगों ने खेती से सम्बलन्धत
 The people of Bhil caste lived in Khandesh
करिनाआयों तथा ऄंग्रेज़ी हुकू मत के डर के कारण
1812-1819 इ. के मध्य लविोह ककया।
situated on the west coast. These people
 यह लविोह 'सेवरम' के नेतृत्व में 1825 इ. में पुनः
revolted between 1812-1819 due to difficulties
ककया गया था।
related to agriculture and fear of British rule.
 भीलों के िारा तीसरा लविोह 1831-1846 इ. के
 The rebellion was repeated in 1825 under the मध्य ककया गया। परन्तु यह लविोह नृशंसता से दबा
leadership of 'Sevaram'. कदया गया।
 The third rebellion by the Bhils took place राजस्थान का भील लविोह
between 1831-1846. But this rebellion was  भील लविोह मेवाड, राजस्थान में हुअ था।
brutally suppressed.  1818 इ. में भीलों के मद्यपान पर लनयन्त्रण एवं
Bhil rebellion of Rajasthan ऄन्धलवश्वास को दूर करने के ललए कु छ सुधार लागू
ककये गए। आसे भीलों ने ऄपनी परम्पराओं को ईल्लंघन
 The Bhil rebellion took place in Mewar,
समझा और लविोह कर कदया एवं 1821 में दौलत हसह
Rajasthan.
के नेतृत्व में ईपिव करना शुरू ककया।
 In 1818, some reforms were implemented to  वषत 1921-1922 में मोतीलाल तेजावत के नेतृत्व में
control the drinking of Bhils and remove इडर, डू ंगरपुर, लसरोही एवं दाता अकद स्थानों में भील
superstition. The Bhils considered it a violation of अन्दोलन पुन: प्रारम्भ हो गया। भीलों ने सम्पूणत भील
their traditions and revolted and in 1821 started क्षेत्र में एक ही पद्धलत से भू-राज एकलत्रत करने की
मााँग की।
rioting under the leadership of Daulat Singh.
 In the year 1921-1922, under the leadership of
Motilal Tejawat, the Bhil movement started again
in places like Idar, Dungarpur, Sirohi and Data.
The Bhils demanded the collection of land-raj in
the entire Bhil region by a single method.

65. Answer. (d) ईत्तर. (d)


 Canning established the Universities of Calcutta,  कै हनग ने लंदन लवश्वलवद्यालय की तजत पर 1857 में
Madras, and Bombay in 1857 on the lines of the कलकत्ता, मिास, और बम्बइ लवश्वलवद्यालय की
University of London. स्थापना की।

©MakeIAS www.makeias.in 9899282107, 8700476287


66. Answer. (c) ईत्तर. (c)
Munda rebellion मुंडा लविोह
 This rebellion was carried out in Jharkhand  ये लविोह 1899-1901 के मध्य झारखण्ड में चलाया
between 1899-1901, which is called Munda गया लजसे मुंडा ईलगुलन या महान हलचल कहा जाता
है।
Ulgulan or Great Movement.
 परं परागत रूप से प्रचललत सामूलहक कृ लष (खून्टकु ट्टी)
 This rebellion took place due to the declaration of
को जागीरदारों , िे केदारों, बलनयों, सूदखोरों के िारा
the traditionally prevalent collective agriculture लनजी संपलत्त घोलषत करने के कारण ही यह लविोह
(Khuntkutti) as private property by the jagirdars, हुअ।
contractors, banias, usurers.  इसाइ लमशनररयों िारा धमत पररवततन का प्रयास भी
प्रमुख समस्या थी।
 Attempts to convert by Christian missionaries  लविोह क बाद छोटा नागपुर टेनेंसी एरट पास ककया
were also a major problem. गया लजससे मुंडा जाती के ऄलधकारों कों संरक्षण
 After the rebellion, the Chota Nagpur Tenancy लमला।

Act was passed, which protected the rights of the


Munda caste.

67. Answer. (b) ईत्तर. (b)


Tana Bhagat Movement ताना भगत अन्दोलन
 The Tana Bhagat movement started in 1914. I  ताना भगत अन्दोलन की शुरुअत वषत 1914 ईं. में
was born in Bihar. लबहार में हुइ थी।
 It was a religious movement that had political  यह ऐसा धार्षमक अन्दोलन था , लजसके राजनीलतक
लक्ष्य थे।
goals.
 जतरा भगत ने 1914 में अकदवासी समाज में पशु
 In 1914, Jatra Bhagat started a campaign to lead
बलल, मांस भक्षण , जीव हत्या , शराब सेवन अकद
a sattvic life in the tribal society, leaving behind
दुगुतणों को छोड कर सालत्वक जीवन यापन करने का
the bad qualities of animal sacrifice, meat-eating, ऄलभयान छे डा। ईन्होंने भूत-प्रेत जैसे ऄंधलवश्वासों के
animal killing, alcohol consumption, etc. He लख़लाण सालत्वक एवं लनडर जीवन की नयी शैली का
ushered in a new style of sattvik and fearless life सूत्रपात ककया।

against superstitions like ghosts and spirits.

68.Answer. (d) ईत्तर. (d)


Indigo Rebellion नील लविोह
 This movement was carried out in Bengal by the  यह अन्दोलन भारतीयों ककसानों िारा लब्ररटश नील
Indian farmers against the British indigo growers. ईत्पादकों के लख़लाण बंगाल में ककया गया।
 ऄपनी अर्षथक मााँगों के सन्दभत में ककसानों िारा ककया
 This movement by the farmers in the context of
जाने वाला यह अन्दोलन ईस समय का एक लवशाल
their economic demands was a huge movement अन्दोलन था।
of that time.  ऄंग्रेज़ ऄलधकारी बंगाल तथा लबहार के ज़मींदारों से
 The British officers took land from the zamindars भूलम लेकर लबना पैसा कदये ही ककसानों को नील की
खेती में काम करने के ललए लववश करते थे , तथा नील
of Bengal and Bihar and forced the farmers to
ईत्पादक ककसानों को एक मामूली सी रकम ऄलग्रम
work in indigo cultivation without paying any
देकर ईनसे करारनामा ललखा लेते थे , जो बाज़ार भाव

©MakeIAS www.makeias.in 9899282107, 8700476287


money, and by giving a small amount advance to से बहुत कम दाम पर हुअ करता था।
the indigo growers, they used to execute  आस प्रथा को 'ददनी प्रथा' कहा जाता था।
agreements with them, which were much more  1860 इ. के नील लविोह का वणतन 'दीनबन्धु लमत्र' ने
than the market price. This practice was called ऄपनी पुस्तक 'नील दपतण ' में ककया है। आस अन्दोलन
'Dadni Pratha'. की शुरुअत 'कदगम्बर' एवं 'लवष्णु लवश्वास' ने की थी।
 The Indigo rebellion of 1860 has been described 'लहन्दू पैरियट ' के संपादक 'हरीशचन्ि मुखजी ' ने भी
by 'Deenbandhu Mitra' in his book 'Nil Darpan'. नील अन्दोलन में काणी काम ककया।
This movement was started by 'Digambar' and  1860 इ. तक नील की खेती ईक्त क्षेत्र में पूरी तरह
ख़त्म हो गइ।
'Vishnu Vishwas'. Harishchandra Mukherjee, the
editor of 'Hindu Patriot', also did a lot of work in
the Indigo movement.
 By 1860, the cultivation of indigo had completely
ended in the said area.

69. Answer. (a) ईत्तर. (a)


The 'First World War' started in 1914.  1914 में 'प्रथम लवश्वयुद्ध' प्रारम्भ हुअ।
 'Communist Party of India' was formed in 1921 by  'एम.एन.राय' िारा 1921 इ. में 'भारतीय कम्युलनस्ट
'M.N.Roy'. पाटी' का गिन ककया गया।
 In December 1925, the famous Arya Samaj  कदसम्बर, 1925 इ. में प्रलसद्ध अयत समाजी राष्ट्रवादी
nationalist leader 'Swami Sahajanand' was नेता 'स्वामी सहजानंद' की हत्या कर दी गइ।
assassinated.  काकोडी िेन कांड (1925 में)।
 Kakori train incident (in 1925).

70.Answer. (d) ईत्तर. (d)


 The Ramosi rebellion started due to the problem  रामोसी लविोह ऄकाल और भूख की समस्या के चलते
of famine and hunger. प्रारम्भ हुअ था।

 The Ramosis' lived in the Western Ghats who  पलश्चमी घाट में रहने वाले 'रामोसी जालत' के लोगों ने

revolted in 1822 AD under the leadership of their 1822 इ. में ऄपने नेता सरदार 'लचत्तर हसह' के नेतृत्व

leader Sardar 'Chittur Singh'. में यह लविोह ककया।


 रामोलसयों ने सतारा के अस-पास के क्षेत्रों को लूटा
 The Ramosis plundered the areas around Satara
और ककलों पर भी अक्रमण कर कदया।
and also attacked the forts.

71. Answer. (d) ईत्तर. (d)


Diarchy in Bengal बंगाल में िैध शासन
 Dyarchy was imposed in Bengal under the Treaty  िैध शासन बंगाल में 1765 इ. की आलाहाबाद सलन्ध
के ऄंतगतत लगाया गया था। यह शासन बंगाल के
of Allahabad in 1765 AD. Apart from Bengal, this
ऄलतररक्त लबहार और ईडीसा में भी लागू ककया गया
rule was also implemented in Bihar and Orissa. था। सलन्ध के फलस्वरूप एक ओर इस्ट आलण्डया
The treaty ended the war between the East India कम्पनी और दूसरी ओर ऄवध के नवाब शुजाईद्दौला ,
Company on the one hand and Shuja-ud-daula, बंगाल के नवाब मीर कालसम और कदल्ली के सम्राट

©MakeIAS www.makeias.in 9899282107, 8700476287


the Nawab of Awadh on the other, Mir Qasim, the शाहअलम लितीय के बीच युद्ध का ऄन्त हो गया।
Nawab of Bengal, and Shah Alam II, the Emperor
of Delhi.

72. Answer. (b) ईत्तर. (b)


 In the 'Tripuri session' of the Congress, Subhash  कांग्रेस के 'लत्रपुरी ऄलधवेशन' में सुभाषचन्ि बोस पुनः
Chandra Bose was again elected the President of कांग्रेस के ऄध्यक्ष चुने गये , परन्तु गााँधी जी के लवरोध
the Congress, but due to the opposition of के चलते ईन्होंने त्यागपत्र दे कदया तथा ऄप्रैल , 1939
Gandhi, he resigned and in April, 1939, Subhash इ. मे सुभाषचन्ि बोस ने 'णारवडत जलॉक' नाम की एक
Chandra Bose formed a new party named नइ पाटी का गिन ककया , सुभाषचन्ि बोस के त्याग
'Forward Bloc'. After the resignation of Subhash पत्र के पश्चात् डॉ. राजेन्ि प्रसाद को कांग्रेस का ऄध्यक्ष
बनाया गया था।
Chandra Bose, Dr. Rajendra Prasad was made
the President of Congress.

73. Answer. (c) ईत्तर. (c)


Surendranath Banerjee said on Partition that Partition  सुरेन्िनाथ बनजी ने लवभाजन पर कहा कक लवभाजन
हमारे उपर एक वज्र की तरह लगरा है।
has fallen on us like a thunderbolt.

74. Answer. (c) ईत्तर. (c)


 The Indian National Congress appointed a  भारतीय राष्ट्रीय कांग्रेस ने जललयांवाला बाग नृशंस
घटना की जााँच के ललए मदन मोहन मालवीय के
commission under the leadership of Madan
नेतृत्व में एक अयोग लनयुक्त ककया , लजसके ऄन्य
Mohan Malviya to inquire into the Jallianwala
सदस्य थे , पंलडत मोतीलाल नेहरू , गांधी जी, ऄजबास
Bagh massacre, whose other members were
तैय्यब जी, सी.अर. दास एवं पुपुल जयकर।
Pandit Motilal Nehru, Gandhiji, Abbas Tayyabji,
C.R. Das and Pupul Jayakar.

75. Answer. (c) ईत्तर. (c)


Widow remarriage लवधवा पुनर्षववाह
 Acharya Ishwar Chandra Vidyasagar of 'Sanskrit  कलकत्ता के 'संस्कृ त कॉलेज ' के अचायत इश्वरचन्ि
लवद्यासागर ने आस कदशा में ईल्लेखनीय कायत ककया।
College' of Calcutta did remarkable work in this
ईन्होंने लवधवा लववाह के समथतन में लगभग एक सहस्र
direction. He submitted an application in support हस्ताक्षरों वाला प्राथतना पत्र तत्कालीन गवतनर
of widow remarriage to the then Governor डलहौज़ी को कदया , लजसके पररणामस्वरूप 1856 इ.
Dalhousie with about a thousand signatures, as a में ‘लहन्दू लवधवा पुनर्षववाह ऄलधलनयम ’ िारा लवधवा
result of which widow remarriage was recognized लववाह को मान्यता दे दी गइ।
in 1856 by the 'Hindu Widow Remarriage Act'.  आस क्षेत्र में धोंडो के शव कवे एवं वीरे सहलगम पुण्टलु ने
 Dhondo Keshav Karve and Veeresalingam भी ईल्लेखनीय कायत ककया। कवे महोदय ने पूना में
1899 इ. में एक ‘लवधवा अश्रम’ स्थालपत ककया।
Puntalu also did remarkable work in this area.
 ईनके प्रयास से ही 1906 इ. में बम्बइ में 'भारतीय
Karve established a 'widow ashram' in 1899 AD
मलहला लवश्वलवद्यालय ' की स्थापना की गइ। धोंडो
in Poona.
के शव कवे लवधवा पुनर्षववाह संघ के सलचव थे।

©MakeIAS www.makeias.in 9899282107, 8700476287


 Due to her efforts, 'Indian Women's University'
was established in Bombay in 1906. Dhondo
Keshav Karve was the secretary of the Widow
Remarriage Association.

76. Answer. (c) ईत्तर. (c)


 Under the Civil Marriage Act (1872 AD), the  लसलवल मैररज एरट (1872 इ.) के ऄन्तगतत 14 वषत से
marriage of girls below the age of 14 years and कम अयु की कन्याओं तथा 18 वषत से कम अयु के
boys below the age of 18 years was prohibited. लडकों का लववाह वर्षजत कर कदया गया। आस
ऄलधलनयम के िारा बहुपत्नी प्रथा भी समाप्त कर दी
Polygamy was also abolished by this act.
गयी।

77.Answer. (a) ईत्तर. (a)


 Ghadar Party - Lala Hardayal - 1913  गदर पाटी - लाला हरदयाल - 1913
 India Women's Association - Annie Besant - 1917  भारत मलहला संघ - एनी बेसेंट - 1917
 Visva Bharati - Rabindra Nath Tagore - 1918  लवश्व भारती - रवीन्ि नाथ टैगोर - 1918
 Forward Bloc - Subhash Chandra Bose - 1939  फॉरवडत जलाक - सुभाष चंि बोस - 1939
 Azad Hind Fauj - Mohan Singh - 1942  अजाद हहद णौज - मोहन हसह - 1942

78.Answer. (a) ईत्तर. (a)


Ryotwari system रै य्यतवाडी व्यवस्था
 भू-राजस्व प्रबंधन के लवषय में लब्ररटश भारत में लागू
 This was the second system of land revenue
की गयी स्थायी बंदोबस्त के बाद यह दूसरी व्यवस्था
management after the Permanent Settlement in
थी, लजसके सूत्रधार थामस मुनरो और कै प्टन रीड थे।
British India, which was pioneered by Thomas
 1792 इ. में कै प्टन रीड ने रै य्यतवाडी व्यवस्था को
Munro and Captain Reed. सवतप्रथम तलमलनाडु के बारामहल लजले में लागू ककया।
 In 1792, Captain Reed implemented the Ryotwari  यह व्यवस्था लब्ररटश भारत के 51 % भाग (मिास
system for the first time in Baramahal district of बंबइ के कु छ लहस्से , पूवी बंगाल, ऄसम, कु गत) पर लागू
Tamil Nadu. की गइ।
 This system was implemented on 51% of British  आस व्यवस्था के ऄंतगतत रै य्यतों को भूलम का
माललकाना हक कदया गया, लजसके िारा ये प्रत्यक्ष रूप
India (Madras parts of Bombay, East Bengal,
से सीधे या व्यलक्तगत रूप से भू-राजस्व ऄदा करने के
Assam, Coorg). ललए ईत्तरदायी थे।
 Under this system, the ryots were given the title  आस पद्धलत में ककसान को 33 % से 55 % के बीच
of land, by which they were directly or personally लगान कं पनी को ऄदा करना होता था। 1836 इ. के
liable to pay the land revenue. बाद हवगेर और गोल्डलस्मथ िारा आस व्यवस्था में
 In this method, the farmer had to pay between सुधार ककए गए।

33% to 55% of the rent to the company. After


1836, this system was improved by Winger and
Goldsmith.

79. Answer. (b) ईत्तर. (b)

©MakeIAS www.makeias.in 9899282107, 8700476287


 Ghadar Party - Lala Hardayal - 1913  गदर पाटी - लाला हरदयाल - 1913
 Women's Indian Association - Annie Besant -  भारत मलहला संघ - एनी बेसेंट - 1917
1917  लवश्व भारती - रवीन्ि नाथ टैगोर - 1918
 Visva Bharati - Rabindra Nath Tagore - 1918  फॉरवडत जलाक - सुभाष चंि बोस - 1939
 Forward Bloc - Subhash Chandra Bose - 1939  अजाद हहद णौज - मोहन हसह - 1942
 Azad Hind Fauj - Mohan Singh - 1942

80. Answer. (b) ईत्तर. (b)


 Asiatic Society of Bengal - William Jones - 1784  एलशयारटक सोसाआटी ऑफ बंगाल - लवललयम जोंस -
 Sanskrit College - Jonathan Duncan - 1792 1784
 Land Holders Society - Dwarka Nath Tagore -  संस्कृ त कॉलेज - जोनाथन डंकन - 1792
1838  लैंड होल्डसत सोसाआटी - िारका नाथ टैगोर - 1838
 Tattvabodhini Sabha - Devendra Nath Tagore -  तत्वबोधनी सभा - देवेन्ि नाथ टैगोर - 1839
1839

81 Answer. (d) ईत्तर. (d)


 Radha Swami Satsang - Swami Shiv Dayal -  राधा स्वामी सत्संग - स्वामी लशव दयाल - 1861
1861  इस्ट आं लडया एसोलसएशन - दादा भाइ नौरोजी -
 East India Association - Dadabhai Naoroji-1866 1866

 Indian Association - Surendra Nath Banerjee -  आं लडयन एसोलसएशन - सुरेन्ि नाथ बनजी - 1876

1876  देव समाज - लशवनारायण ऄलग्नहोत्री - 1887

 Dev Samaj - Shivnarayan Agnihotri - 1887

82. Answer. (a) ईत्तर. (a)


 The All India Hindu Mahasabha was founded in  ऄलखल भारतीय हहदू महासभा की स्थापना 1909 में
1909 to meet the challenges of the Muslim मुलस्लम लीग की चुनौलतयों का सामना करने के ललए
की गइ थी। अरं भ में आसका नाम पंजाब हहदू सम्मेलन
League. Initially its name was Punjab Hindu
था।
Conference.
 1915 में आसका नाम बदलकर हहदू महासभा रखा
 In 1915 it was renamed as Hindu Mahasabha
गया तथा 1921 में भारतीय हहदू महासभा हो गया।
and in 1921 became Bharatiya Hindu
 पंलडत मदन मोहन मालवीय को आसका संस्थापक
Mahasabha. माना जाता है।
 Pandit Madan Mohan Malviya is considered to be
its founder.

83. Answer. (b) ईत्तर. (b)


 Swami Vivekananda, the founder of the  रामकृ ष्ण लमशन के संस्थापक स्वामी लववेकानंद का
Ramakrishna Mission, was born on 12 January जन्म 12 जनवरी 1863 को कोलकाता में हुअ था।
 आनके बचपन का नाम नरें ि नाथ दत्त था।
1863 in Kolkata.

©MakeIAS www.makeias.in 9899282107, 8700476287


 His childhood name was Narendra Nath Dutt.  आन्हें स्वामी लववेकानंद नाम खेतडी के महाराजा ने
कदया था।
 He was given the name Swami Vivekananda by
 स्वामी लववेकानंद ने 1893 में ऄमेररका के लशकागो
the Maharaja of Khetri.
शहर के लवश्व धमत सम्मेलन में भाग ललया , जहां
 Swami Vivekananda participated in the World
ईन्होंने हहदू धमत पर भाषण देकर भारतीय सभ्यता एवं
Conference of Religions in Chicago, USA in संस्कृ लत को गौरवालन्वत ककया। आसके पश्चात
1893, where he glorified Indian civilization and लववेकानंद ने ऄमेररका में वेदांत सभा की स्थापना भी
culture by giving a speech on Hinduism. After this की।
 ऄमेररका से भारत लौटने के बाद ईन्होंने रामकृ ष्ण
Vivekananda also established the Vedanta लमशन की स्थापना की।
Sabha in America.
 After returning to India from America, he
established the Ramakrishna Mission.

84. Answer. (a) ईत्तर. (a)


 The original name of Dayanand Saraswati was  दयानंद सरस्वती का मूल नाम मूल शंकर था। आनका
Mool Shankar. He was born in 1824 in Maurvi, जन्म 1824 में गुजरात के मौरवी में हुअ।
 आन्होंने मथुरा के स्वामी लवरजानंद से वैकदक धमत एवं
Gujarat.
दशतन का शुद्ध रूप से ज्ञान प्राप्त ककया।
 He received pure knowledge of Vedic religion
 दयानंद सरस्वती ने सत्याथत प्रकाश (1874, संस्कृ त)
and philosophy from Swami Virjanand of ग्रंथ में ऄपने लवचार का प्रलतपादन ककया।
Mathura.  वेलेंटाआन लचरौल ने ऄपनी पुस्तक आं लडयन ऄनरे स्ट में
 Dayanand Saraswati presented his idea in the अयत समाज को भारतीय ऄशांलत का जन्मदाता कहा है
 भारतीय हचतकों में स्वराज शजद का प्रयोग सवतप्रथम
book Satyarth Prakash (1874, Sanskrit). दयानंद सरस्वती ने ही ककया था।
 Valentine Chirol in his book Indian Unrest has
called Arya Samaj the father of Indian unrest.
 The word Swaraj was first used by Dayanand
Saraswati among Indian thinkers.

85. Answer. (b) ईत्तर. (b)


 The Young Bengal Movement was founded in the  यंग बंगाल अन्दोलन की स्थापना वषत 1828 इ. में
year 1828 in Bengal. Its founder was 'Henry बंगाल में की गइ थी। आसके संस्थापक 'हेनरी लवलवनय
Vivinay Derozio' (1809-1831 AD). डेरोलजयो' (1809-1831 इ.) थे।
 The main objective of this movement was the  आस अन्दोलन का मुख्य ईद्देश्य प्रेस की स्वतन्त्रता ,
freedom of the press, protection of the ryot from ज़मींदारों िारा ककये जा रहे ऄत्याचारों से रै य्यत की
the atrocities being committed by the landlords, संरक्षा, सरकारी नौकररयों में उाँचे वेतनमान के

and getting Indian people jobs under high pay ऄन्तगतत भारतीय लोगों को नौकरी कदलवाना था।

scales in government jobs.  एंग्लों-आं लडयन डेरोलजयो ‘लहन्दू कॉलेज ’ में ऄध्यापक

 Anglo-Indian Derozio was a teacher in the 'Hindu थे।


 वे फ़्ााँस की महान क्रांलत से बहुत प्रभालवत थे।
College'.
 He was greatly influenced by the Great

©MakeIAS www.makeias.in 9899282107, 8700476287


Revolution of France.

86. Answer. (b) ईत्तर. (b)


 Surendranath Banerjee is the author of the book  आलण्डया ए नेशन नामक पुस्तक के लेखक सुरेन्िनाथ
India A Nation. बैनजी हैं।

87. Answer. (a) ईत्तर. (a)


 The tradition of Brahmo Samaj of Raja  राजा राममोहन राय की ब्रह्म समाज की परं परा को
Rammohun Roy was carried forward by 1843 के बाद देवेन्िनाथ िाकु र ने अगे बढाया। 1866
के बाद आस अंदोलन को के शवचन्ि सेन ने अगे जारी
Devendranath Thakur after 1843. After 1866, this
रखा।
movement was continued by Keshav Chandra  ब्रह्म समाज का प्रभाव ऄलधकांश नगरीय लशलक्षत वगत
Sen. तक ही सीलमत रहा। कफर भी 19वीं तथा 20वीं सदी
 The influence of the Brahmo Samaj was mostly में बंगाल एवं शेष भारत के बौलद्धक , सामालजक,
confined to the urban educated class. सांस्कृ लतक तथा राजनीलतक जीवन पर आसका गहरा
Nevertheless, it had a profound impact on the प्रभाव पडा।

intellectual, social, cultural and political life of


Bengal and the rest of India in the 19th and 20th
centuries.

88.Answer. (c) ईत्तर. (c)


 The socio-religious reform in Bombay province  बंबइ प्रांत में सामालजक-धार्षमक सुधार कायत का अरं भ
was started by Paramhansa Mandali in 1840. 1840 में परमहंस मंडली ने अरं भ ककया।
 आसका मुख्य ईद्देश्य मूर्षतपूजा तथा जालत प्रथा का
 Its main purpose was to oppose idol worship and
लवरोध करना था।
the caste system.
 पलश्चम भारत के पहले धार्षमक सुधारक संभवतः
 The first religious reformer of Western India was
गोपाल हरर देशमुख थे , लजन्हें जनता "लोकलहतवादी"
probably Gopal Hari Deshmukh, whom the public
कहती थी।
called a "Lokhitwadi".

89. Answer. (d) ईत्तर. (d)


 Prarthana Samaj was founded by Atmaram  "प्राथतना समाज" की स्थापना के शवचन्ि सेन की प्रेरणा
Pandurang in Bombay province in 1867 with the से 1867 में अत्माराम पांडुरंग ने बंबइ प्रांत में की।
inspiration of Keshav Chandra Sen.  चूाँकक के शवचन्ि सेन की प्रेरणा से डॉ. अत्माराम
पांडुरंग तथा महादेव गोहवद रानाडे ने प्राथतना समाज
 Since Prarthana Samaj was founded by Dr.
की स्थापना की थी आसललये आस पर ब्रह्म समाज का
Atmaram Pandurang and Mahadev Govind गहरा प्रभाव था।
Ranade with the inspiration of Keshav Chandra  प्राथतना समाज ने एक इश्वर की पूजा का प्रचार ककया
Sen, therefore Brahmo Samaj had a deep तथा धमत को जालत प्रथा की रूकढयों से और पुरोलहतों
के वचतस्व से मुक्त करने का प्रयास ककया।
influence on it.  तेलुगु सुधारक वीरे शहलगम के प्रयासों से प्राथतना
 Prarthana Samaj propagated the worship of one समाज का प्रसार दलक्षण भारत में हुअ।

©MakeIAS www.makeias.in 9899282107, 8700476287


God and tried to free the religion from the
stereotypes of the caste system and the
domination of the priests.
 Prarthana Samaj spread in South India due to the
efforts of Telugu reformer Veeresalingam.

90.Answer. (b) ईत्तर. (b)


Govind Ballabh Pant: गोहवद बल्लभ पंत:
 ईत्तर प्रदेश के कृ लष सुधार पर अधाररत पंत ररपोटत
 Presented the Pant report based on the
प्रस्तुत की।
agricultural reforms of Uttar Pradesh.
 1937 में ईत्तर प्रदेश में कांग्रेस की सरकार बनाइ।
 In 1937, the Congress government was formed in  स्वतंत्र भारत में ईत्तर प्रदेश के प्रथम मुख्यमंत्री बने।
Uttar Pradesh.  सन 1957 में ईन्हें भारतरत्न से सम्मालनत ककया गया।
 Became the first Chief Minister of Uttar Pradesh
in independent India.
 In 1957, he was awarded the Bharat Ratna.

91. Answer. (c) ईत्तर. (c)


Dadabhai Naoroji: दादा भाइ नौरोजी:
 He was popularly known as The Grand Oldman  यह द ग्रैंड ओल्डमैन ऑफ आं लडया के नाम से प्रलसद्ध थे।
of India.  1851 में आन्होंने रहनुमाइ मजदयासन सभा की

 In 1851, he founded the Rehnumai Mazdayasan स्थापना की।

Sabha.  1854 में रश्तगोफ्तार पलत्रका का प्रकाशन ककया।

 In 1854, Rast Goftar magazine was published.  1865 में डजल्यू.सी.बनजी के साथ लमलकर लन्दन

 Founded the London India Society in 1865 along आं लडया सोसायटी की स्थापना की।

with W.C. Banerjee.  1892 में ललबरल पाटी के रटकट पर लब्ररटश हाईस
ऑफ कॉमंस में चुने जाने वाले प्रथम भारतीयथे।
 He was the first Indian to be elected to the British
 1901 में पावटी एवं ऄनलब्ररटश रूल आन आलण्डया
House of Commons in 1892 on a Liberal Party
नामक लनबन्ध ललखा लजसमें धन के बलहगतमन के
ticket.
लसद्धांत का प्रलतपादन ककया।
 Wrote an essay called Poverty and Un-British
Rule in India in 1901 in which he presented the
Drain of wealth theory.

92. Answer. (d) ईत्तर. (d)


Other Education Commissions in British India लब्ररटश भारत में ऄन्य लशक्षा अयोग
 Charles Wood Dispatch - 1854 - Lord Dalhousie  चाल्सत वुड लडस्पैच - 1854 - लाडत डलहौजी

 Hunter Commission - 1882 - Lord Ripon  हंटर अयोग - 1882 - लॉडत ररपन

 Rail Commission - 1902 - Lord Curzon  रै ल अयोग - 1902 - लॉडत कजतन

 Sadler Commission - 1917 - Lord Chelmsford  सैडलर अयोग - 1917 - लॉडत चेम्सफोडत

 Inchcap Commission - 1923 - Lord Reading  आं चके प अयोग - 1923 - लॉडत रीहडग

©MakeIAS www.makeias.in 9899282107, 8700476287


 Hartog Committee (Elementary Education) - 1929  हटोग सलमलत (प्राथलमक लशक्षा)- 1929 - लॉडत आर्षवन
- Lord Irwin  हलडसे अयोग (प्रौढ लशक्षा) - 1929 - लॉडत आर्षवन
 Lindsay Commission (Adult Education) - 1929 -  साजेट अयोग - 1944 - लॉडत वेवेल
Lord Irwin
 Sargent Commission - 1944 - Lord Wavell

93. Answer. (b) ईत्तर. (b)


 See Explanation to question no. 92.  प्रश्न संख्या 92 की व्याख्या देंखें।

94.Answer. (c) ईत्तर. (c)


 The main feature of the religious reform  अधुलनक युग के धार्षमक सुधार अंदोलनों की मुख्य
बात यह थी कक ये अंदोलन बुलद्धवाद तथा
movements of the modern era was that these
मानवतावाद के लसद्धांतों पर अधाररत थे , हालााँकक
movements were based on the principles of
लोगों को ऄपनी ओर खींचने के ललये कभी-कभी
rationalism and humanism, although sometimes अस्था तथा प्राचीन ग्रंथों का सहारा भी लेते थे।
they also resorted to faith and ancient texts to  आन धार्षमक अंदोलनों ने ईभरते हुए मध्यवगत तथा
attract people. अधुलनक लशक्षा प्राप्त प्रबुद्ध लोगों को सबसे ऄलधक
प्रभालवत ककया। आन अंदोलनों ने भारतीय धमों के
 These religious movements most affected the
कमतकांडी, ऄंधलवश्वासी, बुलद्धलवरोधी तथा पुराणपंथी
emerging middle class and modern educated पक्षों का लवरोध ककया।
enlightened people. These movements opposed
the ritualistic, superstitious, anti-intellectual and
orthodox aspects of Indian religions.

95. Answer. (d) ईत्तर. (d)


 The British rule gave rise to many such forces  लब्ररटश शासन ने ऐसी ऄनेक शलक्तयों को जन्म कदया
लजन्होंने धीरे -धीरे जालत की जडों को कमज़ोर ककया।
which gradually weakened the roots of the caste.
अधुलनक ईद्योगों, रे लों व बसों के अरं भ ने तथा बढते
The introduction of modern industries, railways
नगरीकरण के कारण खासकर शहरों में लवलभन्न
and buses and increasing urbanization made जालतयों के लोगों के बीच संपकत को ऄपररहायत बना
contact between people of different castes कदया। अधुलनक व्यापार-ईद्योग ने अर्षथक कायतकलाप
inevitable, especially in cities. Modern trade- के नए क्षेत्र सभी के ललये पैदा ककये। एक अधुलनक
औद्योलगक समाज में जालत और व्यवसाय का पुराना
industry created new areas of economic activity संबंध चलाना करिन था।
for all. It was difficult to maintain the old  प्रशासन के क्षेत्र में , ऄंग्रेज़ों ने कानून के सामने सबकी
relationship of caste and occupation in a modern समानता का लसद्धांत लागू ककया , जालतगत पंचायतों
industrial society. से ईनके न्यालयक काम छीन ललये गए और प्रशासकीय
 In the field of administration, the British सेवाओं के दरवाजे सबके ललये खोल कदये गए। आसके
ऄलावा नइ लशक्षा प्रणाली पूरी तरह धमतलनरपेक्ष थी
implemented the principle of equality before the
आसललये वह मूलतः जालतगत भेदों तथा जालतगत
law, the judicial functions were taken away from दृलिकोण की लवरोधी थी।
the caste panchayats and the doors of
administrative services were opened to all. Apart
from this, the new education system was

©MakeIAS www.makeias.in 9899282107, 8700476287


completely secular, so it was basically opposed
to caste distinctions and caste outlook.

96. Answer. (d) ईत्तर. (d)


 Native Opinion - English - Bombay - B. N.  नेरटव ओलपलनयन - ऄंग्रेजी - बम्बइ - बी. एन.
मांडललक
Mandlik
 बंगाली - ऄंग्रेजी - कलकत्ता - सुरेन्िनाथ बनजी
 Bengalee - English - Calcutta - Surendranath  बम्बइ दपतण - मरािी - बंबइ - बाल शािी
Banerjee  कॉमन वील - ऄंग्रेजी - एनी बेसेंट
 Bombay Darpan - Marathi - Bombay - Bal Shastri
 Common Weal - English - Annie Besant

97. Answer. (c) ईत्तर. (c)


 Dr. Bhimrao Ambedkar, who himself belonged to  डॉ.भीमराव ऄम्बेडकर जो खुद एक ऄनुसूलचत जालत
a Scheduled Caste, devoted his whole life to the के थे , ईन्होंने ऄपना पूरा जीवन जालतगत ऄत्याचार
लवरोधी संघषत को समर्षपत कर कदया। आसके ललये
struggle against caste atrocities. For this he
ईन्होंने ‘ऄलखल भारतीय ऄनुसूलचत जालत महासंघ’ की
founded the 'All India Scheduled Caste
स्थापना की।
Federation'.

98. Answer. (b) ईत्तर. (b)


 Young India, New Life, Harijan - Mahatma  यंग आं लडया, नव जीवन, हररजन - महात्मा गााँधी
Gandhi  आं लडपेंडेस - मोतीलाल नेहरू
 Independence - Motilal Nehru

99. Answer. (b) ईत्तर. (b)


 Born in a lower caste in Maharashtra in the late  महाराष्ट्र में 19वीं सदी के ईत्तराद्धत में लनचली जालत में
19th century, Jyotiba Phule led a lifelong जन्मे ज्योलतबा फु ले ने ब्राह्मणों की धार्षमक सत्ता के
लखलाफ जीवन भर अंदोलन चलाया। वे अधुलनक
movement against the religious authority of
लशक्षा को लनचली जालतयों की मुलक्त का सबसे
Brahmins. He considered modern education to शलक्तशाली ऄि समझते थे। वे पहले व्यलक्त थे लजन्होंने
be the most powerful weapon for the लनचली जालतयों की लडककयों के ललये ऄनेक स्कू ल
emancipation of the lower castes. He was the खोले।

first to open many schools for girls from lower


castes.

100. Answer. (b) ईत्तर. (b)


 Kesari (Marathi) , Maratha (English) - Bombay -  के सरी (मरािी) ,मरािा (ऄंग्रेजी) - बम्बइ - लतलक
Tilak (Initially in collaboration with Agarkar) (प्रारं भ में ऄगरकर के सहयोग से)

101.Answer (a) ईत्तर. (a)

Prime regions Leaders British officers प्रमुख कें ि लविोह के लब्ररटश दमनकतात
नेतृत्वकतात

©MakeIAS www.makeias.in 9899282107, 8700476287


Delhi Bahadur Shah John कदल्ली बहादुर शाह जॉन लनकोलसन ,
Zafar,Bakht Nicolson,Hudson जफर, बख्त खां हडसन
Khan लखनउ बेगम हजरत महल हेनरी लॉरें स ,
कैं पबेल
Lucknow Begum Hazarat Henry
Mahal Lawrence,Camp कानपुर नाना साहब , कैं पबेल
bell तात्या टोपे

Kanpur Campbell झांसी, रानी लक्ष्मीबाइ , जनरल ह्यूरोज

Nanasaheb,Tan ग्वाललयर तात्या टोपे

tya tope जगदीशपुर कुं वर हसह , ऄमर मेजर लवललयम


(लबहार) हसह टेलर, वेसेंट अयर
Jhansi, Gwalior Rani Lakshmi General Hugh
bai,Tantya Tope Rose फै जाबाद मौलवी कैं पबेल
ऄहमदुल्लाह
Jagdishpur(Biha Kunwar Major William
आलाहाबाद, ललयाकत ऄली कनतल नील
r) Singh,Amar taylor,Versent
बनारस
Singh Eyre
बरे ली खान बहादुर खां कैं पबेल
Faizabad Maulvi Campbell
Ahmadullah

Allahabad,Bana Liyaquet Ali Colonel Neil


ras

Bareilly Khan Bahadur Campbell


Khan

102. Answer. (d) ईत्तर. (d)


 See Explanation to question no. 101.  प्रश्न संख्या 101 की व्याख्या देखें।

103. Answer(a) ईत्तर. (a)


 See explanation of Q101  प्रश्न संख्या 101 की व्याख्या देखें।

104. Answer. (a) ईत्तर. (d)


Answer. (d)  नेशन आन मेककग - सुरेन्िनाथ बनजी
 पाककस्तान एंड द पाटीशन ऑफ आं लडया - बी. अर.
 Nation in Making - Surendranath Banerjee
ऄंबेडकर
 Pakistan and the Partition of India - B. R.  भवानी मंकदर - बारीन्ि कु मार घोष
Ambedkar  द कफलॉसफी ऑफ द बॉम्ब - भगवती चरण बोहरा
 Bhavani Temple - Barindra Kumar Ghosh
 The Philosophy of the Bomb - Bhagwati Charan

©MakeIAS www.makeias.in 9899282107, 8700476287


Bohra

105. Answer. (c) ईत्तर. (c)


Books related to the Revolt of 1857: 1857 के लविोह से संबंलधत पुस्तकें :
 The Indian War of Independence 1857: V D  द आं लडयन वॉर ऑफ आं लडपेंडेंस 1857: वी डी सावरकर
Savarkar  द पीजेंट एंड द राज: एररक थॉमस स्टॉरस
 The Peasant and the Secret: Eric Thomas Stocks  1857 द ग्रेट ररवॉल्ट: ऄशोक मेहता
 1857 The Great Revolt: Ashok Mehta  द फस्टत आं लडयन वॉर ऑफ आं लडपेंडेंस: कालत मारसत
 The First Indian War of Independence: Karl Marx और फ्े डररक एंजेल्स
and Friedrich Engels

106. Answer. (c) ईत्तर. (c)


1885 Bombay Session of INC - Womesh Chandra 1885.- बम्बइ - व्योमेश चन्िd बनजी
Banerjee  72 प्रलतलनलधयों ने भाग ललया तथा कांग्रेस के ईद्देश्य
 72 delegates participated and the objectives of तय ककए गए।
the Congress were decided. 1886 - कलकत्ता - दादाभाइ नौरोजी
 यह प्रथम गैर हहदू ऄध्यक्ष थे।
1886 - Calcutta - Dadabhai Naoroji
1887- मिास - बदरुद्दीन तैयब जी
 He was the first non-Hindu president.
 यह प्रथम मुलस्लम ऄध्यक्ष थे।
1887 - Madras - Badruddin Tyabji
1888 - आलाहाबाद - जॉजत यूल
 He was the first Muslim president.  यह प्रथम ऄंग्रेज ऄध्यक्ष थे।
1888 - Allahabad - George Yule 1889- बम्बइ - सर लवललयम वेडरबनत
 He was the first British President.
1889 - Bombay - Sir William Wedderburn

107. Answer. (a) ईत्तर. (a)


1901 - Calcutta - Dinshaw Idulji Wacha 1901 - कलकत्ता - कदनशा आदुलजी वाचा
 Mahatma Gandhi attended the Congress session  आस ऄलधवेशन में महात्मा गांधी कांग्रेस के मंच पर
. अए।
 ईनका ईद्देश्य दलक्षण ऄफ्ीका में अंदोलन के ललए
 His aim was to solicit support for the movement
समथतन मांगना था।
in South Africa.

108. Answer. (a) ईत्तर. (a)


1905 - Banaras - Gopal Krishna Gokhale 1905 - बनारस - गोपाल कृ ष्ण गोखले
 The Partition of Bengal and the reactionary policy  बंगाल लवभाजन और लॉडत कजतन की प्रलतकक्रयावादी
नीलत की हनदा की गइ।
of Lord Curzon were condemned.

109. Answer. (c) ईत्तर. (c)


 1907 - Surat - Dr. Rash Behari Ghosh 1907 - सूरत - डॉ. रास लबहारी घोष
 Congress split.  कांग्रेस का लवभाजन हुअ।
 Tribhuvandas was the chairman of the reception  स्वागत सलमलत के ऄध्यक्ष लत्रभुवनदास थे।

©MakeIAS www.makeias.in 9899282107, 8700476287


committee.

110.Answer- (a) ईत्तर- (a)


 The Sadler Commission was formed in 1917 AD  सैडलर अयोग का गिन 1917 इ. में 'कलकत्ता
to study the problems of 'University of Calcutta' लवश्वलवद्यालय' की समस्याओं के ऄध्ययन के ललए
by Dr. M.E. Sadler's leadership. डॉरटर एम.इ. सैडलर के नेतृत्व में ककया गया था।
 Two Indians, Dr. Ashutosh Mukherjee and Dr.  आस अयोग में दो भारतीय भी , डॉरटर अशुतोष
Ziauddin Ahmed, were also members of this मुखजी एवं डॉरटर लजयाईद्दीन ऄहमद, सदस्य थे।
commission.  आस अयोग ने कलकत्ता लवलश्वद्यालय के साथ-साथ
माध्यलमक स्नातकोत्तरीय लशक्षा पर भी ऄपना मत
 This commission expressed its opinion on
व्यक्त ककया।
Calcutta University as well as secondary post-
graduate education.

111. Answer. (d) ईत्तर. (d)


 1937 AD - Faizpur - Jawaharlal Nehru  1937 इ.-णै जपुर - जवाहर लाल नेहरु

112.Answer. (c) ईत्तर. (c)


 Lord Dufferin has said that Congress represents  लॉडत डफररन ने कहा है कक कांग्रेस जनता के ईस
ऄल्पसंख्यक वगत का प्रलतलनलधत्व करती है लजसकी
that minority section of the people whose
संख्या सूक्ष्म है।
numbers are small.  लॉडत कजतन ने कहा है कांग्रेस ऄपनी मौत की घलडयां
 Lord Curzon has said that the Congress is लगन रही है , भारत में रहते हुए मेरी तीव्र आच्छा है कक
counting the clocks of its death, while living in मैं ईसे शांलतपूवतक मरने में सहायता करूं। साथ ही ,
India I have a strong desire to help it die आसमें कांग्रेस को देशिोही संगिन कहा है।
peacefully. Also, in this, Congress has been  लाला लाजपत राय ने कांग्रेस को लॉडत डफररन के
कदमाग की ईपज बताया है और आसे लशलक्षत
called an anti-national organization.
भारतीयों के वार्षषक राष्ट्रीय मेले की संज्ञा दी है।
 Lala Lajpat Rai has described the Congress as
the brainchild of Lord Dufferin and called it the
annual national fair of educated Indians.

113. Answer. (c) ईत्तर. (c)


 Nations Voice - Radhakrishnan  नेशन्स वॉयस - राधाकृ ष्णन
 Gathering Storm - Lapierre, Collins  गैदटरग स्टॉमत - लैलपयरे, कॉहलस
 एसेज़ ऑन गीता - ऄरहवद घोष
 Essays on Geeta - Aravind Ghosh
 नील दपतण - दीनबंधु लमत्र
 Neel Darpan - Deenbandhu Mitra

114. Answer. (d) ईत्तर. (d)


 The Scope of Happiness - Vijayalakshmi Pandit  द स्कोप ऑफ हैप्पीनेस - लवजयलक्ष्मी पंलडत
 आं लडयाज पास्ट - अथतर ए. मैरडॉनल
 India's Past - Arthur A. mcdonnell
 आं लडया एंड आं लडयन लमशन - एलेरजेंडर डफ
 India and Indian Mission - Alexander Duff  भारतीय स्वतंत्रता संग्राम - वी. डी. सावरकर
 Indian Freedom Struggle - VD Savarkar

©MakeIAS www.makeias.in 9899282107, 8700476287


115.Answer. (b) ईत्तर. (b)
East India Association इस्ट आं लडया एसोलसएशन
 Dadabhai Naoroji founded the East India  दादाभाइ नौरोजी ने 1 ऄरटू बर 1866 को लंदन में
इस्ट आं लडया एसोलसएशन की स्थापना की।
Association in London on 1 October 1866.
 तत्कालीन भारतीय समस्याओं पर लवचार करना तथा
 To ponder over Indian problems of the time and लब्ररटश जनमत को प्रभालवत करना।
influence British public opinion.

116. Answer. (d) ईत्तर. (d)


Madras Mahajan Sabha-1884(Madras) मिास महाजन सभा-1884(मिास)
 Established by - M Veeraraghavachari, B  स्थापना - एम वीरराघवाचारी , बी सुब्रह्मण्य ऄय्यर ,
Subrahmanya Iyer, P Ananda Charlu पी अनंद चालूत
 Purpose - To coordinate the work of local  ईद्देश्य - स्थानीय संगिनों व संस्थाओं के कायो को
समलन्वत करना।
organizations and institutions.

117.Answer. (a) ईत्तर. (a)


 Unhappy India - Lala Lajpat Rai  ऄनहैप्पी आं लडया - लाला लाजपत राय
 गीता रहस्य - बाल गंगाधर लतलक
 Geeta Rahasya - Bal Gangadhar Tilak
 बंदी जीवन - शचींि नाथ सान्याल
 Bandi Jeevan - Sachindra Nath Sanyal  आं लडयन होमरूल - महात्मा गांधी
 Indian Home Rule - Mahatma Gandhi

118. Answer. (d) ईत्तर. (d)


 The Ilbert Bill was introduced during the tenure of  वायसराय लॉडत ररपन के कायतकाल के दौरान आल्बटत
the Viceroy Lord Ripon. लबल लाया गया था।
 आस लबल के िारा भारतीय न्यायाधीशों को ईन
 By this bill, Indian judges were also empowered
मामलों की सुनवाइ करने का भी ऄलधकार प्रदान कर
to hear cases in which European citizens were कदया गया लजनमें यूरोपीय नागररक भी शालमल होते
also involved. थे।
 This is called the Ilbert Bill controversy.  आसे ही आल्बटत लबल लववाद कहा जाता है।

 Due to extreme opposition to this bill, the Viceroy  आस लबल के ऄत्यलधक लवरोध के चलते वायसराय ने
आसे वापस ले ललया था।
withdrew it.

119. Answer. (b) ईत्तर. (b)


 Gitanjali, Gora, Crescent Moon, Post Office -  गीतांजलल, गोरा, क्रीसेंट मून , पोस्ट ऑकफस -
Rabindranath Tagore रवीन्िनाथ टैगोर
 पथेर दावी - शरतचंि चट्टोपाध्याय
 Pather Dabi - Saratchandra Chattopadhyay

120. Answer- (c) ईत्तर- (c)


 First Carnatic War (1746-48) - Treaty of Aix-la-  प्रथम कनातटक युद्ध( 1746-48)- एरसा-ला-शापेल की
Chapelle संलध
 Second Carnatic War (1749-54) - Inconclusive  लितीय कनातटक युद्ध(1749-54)- ऄलनणातयक युध्द
War  तृतीय कनातटक युद्ध( 1758-63)- पेररस की संलध से

©MakeIAS www.makeias.in 9899282107, 8700476287


 Third Carnatic War (1758-63) - End with the ऄंत
Treaty of Paris  प्रथम अंग्ल मैसूर युद्ध(1767-69)- मिास की संलध

 First Anglo-Mysore War (1767-69) - Treaty of  तृतीय अंग्ल मैसूर युद्ध( 1790-92) - श्रीरं गपट्टनम की
संलध
Madras
 Third Anglo-Mysore War (1790-92) - Treaty of
Srirangapatnam

121. Answer. (b) ईत्तर. (b)


Year 1917: President of Calcutta session: Annie Besant,  वषत 1917: कलकत्ता। ऄध्यक्ष: एनी बेसेंट , कॉन्ग्रेस की
the first woman President of Congress पहली मलहला ऄध्यक्ष
Year 1925: President of Kanpur Session: Sarojini Naidu,  वषत 1925: कानपुर। ऄध्यक्ष: सरोलजनी नायडू , पहली
भारतीय मलहला ऄध्यक्ष।
the first Indian woman president.

122. Answer. (a) ईत्तर. (a)


 Revolutionary activities in order to be operated  क्रांलतकारी गलतलवलधयां संचाललत करने के ललये धन
required finance. On 9 August 1925, the की अवश्यकता थी। 9 ऄगस्त 1925 इ॰ में सहारनपुर
लखनउ लाआन पर काकोरी रे लवे स्टेशन पर
revolutionaries succeeded in robbing the
क्रांलतकारी सरकारी खजाने को लूटने में सफल हुए।
government treasury at Kakori railway station on आस घटना से भारी संख्या में युवकों को लगरफ्तार
the Saharanpur-Lucknow line. A large number of ककया गया और ईन पर मुकदमा चला।
youths were arrested and prosecuted for this  ऄशफाक ईल्ला खााँ, रामप्रसाद लबलस्मल, रोशन हसह,
incident. राजेन्ि लालहडी को फााँसी दे दी गइ। 17 लोगों को
 Ashfaq Ulla Khan, Ramprasad Bismil, Roshan लम्बी सजाएं सुनाइ गईं।
Singh, Rajendra Lahiri were hanged. Long
sentences were given to 17 people.

123.Answer. (b) ईत्तर. (b)


 In the second phase of revolutionaries, youth  क्रांलतकाररयों के लितीय चरण में युवाओं ने कइ
क्रांलतकारी संगिन बनाये लजसमें भगतहसह यशपाल
formed many revolutionary organizations in which
तथा छबीलदास ने नौजवान सभा की स्थापना की।
Bhagat Singh Yashpal and Chhabildas founded
 1928 इ॰ में चन्िशेखर अजाद , शचीन्िनाथ सान्याल,
the Naujawan Sabha.
रामप्रसाद लबलस्मल अकद ने लमलकर ‘लहन्दुस्तान
 In 1928, Chandrashekhar Azad, Sachindra Nath
ररपलजलक एसोलसएशन’ का नाम बदल कर ‘लहन्दुस्तान
Sanyal, Ramprasad Bismil etc. together changed
सोशललस्ट ररपलजलक एसोलसएशन ’ रखा। लजसका
the name of 'Hindustan Republican Association'
मुख्य ईद्देश्य क्रांलत के माध्यम से औपलनवेलशक सत्ता
to 'Hindustan Socialist Republican Association'. को ईखाड फें कना था। ऄपने स्वरूप में यह संगिन
Its main objective was to overthrow the colonial लोकतांलत्रक था।
power through revolution. This organization was
democratic in its nature.

124. Answer-(a) ईत्तर-(a)

©MakeIAS www.makeias.in 9899282107, 8700476287


 Individual Satyagraha was started by the Father  व्यलक्तगत सत्याग्रह राष्ट्रलपता महात्मा गााँधी िारा सन
of the Nation Mahatma Gandhi in the year 1940. 1940 में प्रारम्भ ककया गया था।
 आस सत्याग्रह कक सबसे बडी बात यह थी कक आसमें
 The biggest thing about this Satyagraha was that
महात्मा गााँधी िारा चुना हुअ सत्याग्रही पूवत लनधातररत
the Satyagrahi chosen by Mahatma Gandhi gave
स्थान पर भाषण देकर ऄपनी लगरफ्तारी देता था।
his arrest by giving a speech at a predetermined  ऄपने भाषण से पूवत सत्याग्रही ऄपने सत्याग्रह की
place. सूचना लज़ला मलजस्िेट को भी देता था।
 Before his speech, the satyagrahi also informed  लवनोवा भावे प्रथम सत्याग्राही थे।
the district magistrate about his satyagraha.
 Vinova Bhave was the first Satyagrahi.

125. Answer. (b) ईत्तर. (b)


August offer ऄगस्त प्रस्ताव
 In March 1940, Congress passed a resolution in  माचत, 1940 को कांग्रेस ने ऄपने रामगढ (लबहार) में
अयोलजत ऄलधवेशन में एक प्रस्ताव पाररत कर
its session held in Ramgarh (Bihar) and asked
सरकार से कहा कक यकद वह के न्ि में एक ऄंतररम
the government that if it formed an interim
राष्ट्रीय सरकार गरित करे तो , कांग्रेस लितीय लवश्व
national government at the center, Congress युद्ध में सरकार का सहयोग कर सकती है।
could support the government in World War II.  कांग्रेस के आस प्रस्ताव के जवाब में तत्कालीन
 In response to this resolution of the Congress, वायसराय लाडत ललनललथगो ने 8 ऄगस्त, 1940 को
the then Viceroy Lord Linlithgow presented the ऄगस्त प्रस्ताव प्रस्तुत ककया।

August Resolution on August 8, 1940.

126. Answer- (c) ईत्तर- (c)


 Bengal Gazetteer - James Augustus Hickey  बंगाल गजट - जेम्स ऑगस्टस लहक्की
 Samvad Kaumudi - Raja Ram Mohan Roy  संवाद कौमुदी - राजा राम मोहन राय
 Amrit Bazar Patrika - Bal Gangadhar Tilak  ऄमृत बाजार पलत्रक - बाल गंगाधर लतलक
 Kesari - Motilal Nehru  के सरी - मोतीलाल नेहरू

127. Answer. (c) ईत्तर. (c)


 Nawab Abdul Latif Khan is called the father of  बंगाल में मुलस्लम पुनजातगरण का लपता नवाब ऄजदुल
Muslim renaissance in Bengal. लतीण खान को कहा जाता है।

 Nawab Abdul Latif Khan was the pioneer of  नवाब ऄजदुल लतीण खान 19वीं सदी में मुलस्लम

Muslim modernization in the 19th century, his अधुलनकीकरण के पथ प्रदशतक थे , लशक्षा के क्षेत्र में
ईनका योगदान महत्वपूणत था।
contribution in the field of education was
 ईनकी तुलना राजा राम मोहन राय से की जाती है
significant. रयोंकक नवाब ऄजदुल को ऄपने समय का सबसे बडा
 He is compared to Raja Ram Mohan Roy as मुलस्लम सुधारक माना जाता था।
Nawab Abdul was considered the greatest  ईन्होंने ऄप्रैल 1863 में मोहम्मडन ललटरे री सोसाआटी

Muslim reformer of his time. की भी स्थापना की।


 ईन्होंने मदरसे के ललए एक नया पाठ्यक्रम भी
 He also founded the Mohammedan Literary
प्रस्तालवत ककया लजसमें ऄंकगलणत , बीजगलणत,
Society in April 1863.

©MakeIAS www.makeias.in 9899282107, 8700476287


 He also proposed a new curriculum for the ज्यालमलत, नैलतक दशतन के साथ दशतन शालमल थे।
madrassa which included philosophy along with
arithmetic, algebra, geometry, and moral
philosophy.

128. Answer. (c) ईत्तर. (c)


 Khan Abdul Ghaffar Khan started the Khudai  खान ऄजदुल गफ्फार खान ने वषत 1929 में खुदाइ
Khidmatgar (Servant of God) movement in the लखदमतगार (सवेंट ऑफ गॉड) अंदोलन की शुरुअत
की। सामान्य लोगों की भाषा में वे सुखत पोश थे। खुदाइ
year 1929. In the language of the common
लखदमतगर अंदोलन गांधी जी के ऄहहसात्मक
people, he was ruddy-faced. The Khudai अंदोलन से प्रेररत था।
Khidmatgar movement was inspired by Gandhi's  फ्ं रटयर गांधी के नाम से मशहूर ऄजदुल गफ्फार खान
non-violent movement. को बाचा खान और बादशाह खान के नाम से भी जाना
जाता है। महात्मा गांधी के एक दोस्त ने ईन्हें फ्ं रटयर
 Abdul Ghaffar Khan, popularly known as Frontier
गांधी का नाम कदया था।
Gandhi, is also known as Bacha Khan and  यद्यलप लजन्ना के नेतृत्व में ऄलधकतर मुसलमान
Badshah Khan. A friend of Mahatma Gandhi सलवनय ऄवज्ञा अंदोलन से ऄलग रहे , लेककन ईत्तर-
named him Frontier Gandhi. पलश्चमी सीमा प्रांत के पिानों ने खान ऄजदुल गफ्फार
 Although most Muslims under Jinnah's leadership खााँ के नेतृत्व में आस अंदोलन में भाग ललया।

remained aloof from the civil disobedience


movement, the Pathans of the North-West
Frontier Province participated in the movement
under the leadership of Khan Abdul Ghaffar
Khan.

129. Answer. (b) ईत्तर. (b)


 Vanar Sena and Manjari Sena are related to the  वानर सेना और मंजरी सेना का संबंध सलवनय ऄवज्ञा
अंदोलन से है। सलवनय ऄवज्ञा अंदोलन में बच्चों को
Civil Disobedience Movement. In the Civil
लेकर वानर सेना तथा बाललकाओं को लेकर मंजरी
Disobedience Movement, the Vanar Sena was सेना का गिन ककया गया था।
formed for the children and the Manjari Sena was
formed for the girls

130. Answer. (c) ईत्तर. (c)


 The following changes took place in the political लितीय गोलमेज सम्मेलन शुरू होने के िीक पहले आं ग्लैण्ड की
राजनीलतक लस्थलत में लनम्न पररवततन हुए-
situation in England just before the Second
 मजदूर दल की सरकार के स्थान पर राष्ट्रीय सरकार
Round Table Conference began:
का लनमातण हुअ लजसमें मजदूर , ऄनुदार तथा ईदार
 The National Government was formed in place of तीनों दल सलम्मललत हुए.
the Government of the Labor Party, in which all  सर सेमुऄल होर भारत सलचव लनयुक्त ककया गया,
the three workers, liberal and liberal parties  लाडत आर्षवन जैसे ईदारवादी के स्थान पर लॉडत
participated. वेललगंटन वायसराय लनयुक्त हुअ,

 Sir Samuel Hoare was appointed India Secretary,  लितीय सम्मेलन में गााँधीजी ही एक मात्र कांग्रेसी

©MakeIAS www.makeias.in 9899282107, 8700476287


 In place of liberals like Lord Irwin, Lord प्रलतलनलध के रूप में 29 ऄगस्त 1931 को राजपुताना
Wellington was appointed Viceroy. जहाज से लब्रटेन के ललए रवाना हुए। ईनके साथ आस
 In the second conference, Gandhiji was the only वातात में मदन मोहन मालवीय , देवदास गााँधी ,

Congress representative left for Britain on 29 सरोजनी नायडू , एनी बेसेंट , घनश्याम दास लबडला ,
मीरा बेन एवं महादेव देसाइ जैसे लोग भी सलम्मललत
August 1931 by Rajputana ship. People like
थे, जो ऄलग ऄलग दलों से सम्बलन्धत थे।
Madan Mohan Malviya, Devdas Gandhi, Sarojini
Naidu, Annie Besant, Ghanshyam Das Birla, Mira
Ben and Mahadev Desai were also involved in
this dialogue with him, who belonged to different
parties.

131. Answer. (d) ईत्तर. (d)


 On March 11, 1942, Churchill announced that the  11 माचत, 1942 को चर्षचल ने घोषणा की कक भारत
British Government had prepared a plan to के राजनीलतक गलतरोध को दूर करने के ललए लब्ररटश
सरकार ने एक योजना तैयार की है और आस हेतु सर
resolve India's political deadlock and for this Sir
स्टेफोडत कक्रप्स को भारत भेजा जायेगा।
Stafford Cripps would be sent to India.
 स्टेफडत कक्रप्स माचत , 1942 को कदल्ली पहुाँचा। वह
 Stafford Cripps reached Delhi in March 1942. He
कांग्रेस,मुलस्लम लीग , लहन्दू महासभा , हररजनों,
met representatives of Congress, Muslim राजाओं और ऄलतवाकदयों के प्रलतलनलधयों से लमला
League, Hindu Mahasabha, Harijans, Rajas and और तत्पश्चात् 30 माचत, 1942 को ऄपने प्रस्तावों की
extremists and thereafter announced his घोषणा कर दी।
proposals on 30 March 1942.  कांग्रेस और मुलस्लम लीग दोनों ने कक्रप्स प्रस्तावों को
ऄस्वीकार कर कदया।
 Both the Congress and the Muslim League
rejected the Cripps proposals.

132. Answer. (c) ईत्तर. (c)


Quit India Proposal भारत छोडो प्रस्ताव
 It was said in this resolution that if the British  आस प्रस्ताव में कहा गया कक यकद ऄंग्रेज भारत से
ऄपना लनयंत्रण हटा लें तो भारतीय जनता लवदेशी
remove their control from India, then the Indian
अक्रांताओं का सामना करने के ललए हर प्रकार से
people are ready to contribute in every way to योगदान करने को तैयार है।
face the foreign invaders.  आस प्रस्ताव का ऄंलतम लनणतय 7 व 8 ऄगस्त, 1942
 The final decision of this proposal was taken on को बंबइ में कांग्रेस की महासलमलत में ककया गया।
7th and 8th August, 1942 in the General  कांग्रेस महासलमलत ने 8 ऄगस्त, 1942 को भारत
Committee of the Congress in Bombay. छोडो अंदोलन प्रस्ताव कु छ संशोधनों के साथ स्वीकार
 The Congress General Committee accepted the कर ललया।

Quit India Movement resolution on 8 August 1942  आस दौरान गांधी ने करो या मरो का मंत्र कदया।

with some modifications.


 During this, Gandhi gave the mantra of do or die.

133.Answer. (a) ईत्तर. (a)

©MakeIAS www.makeias.in 9899282107, 8700476287


 During the Quit India Movement, all the big  भारत छोडो अंदोलन के दौरान लब्ररटश सरकार िारा
leaders were arrested by the British Government रात को 12 बजे ऑपरे शन ज़ीरो ऑवर (Operation

at 12 o'clock in the night under Operation Zero Zero Hour) के तहत सभी बडे नेता लगरफ्तार कर
ललये गए। महात्मा गांधी को पुणे की अगा खां जेल में
Hour. Mahatma Gandhi was kept in the Aga
रखा गया ।
Khan Jail in Pune.

134. Answer. (b) ईत्तर. (b)


 All India Forward Bloc is a nationalist political  ऑल आं लडया फावतडत जलाक भारत का एक राष्ट्रवादी
राजनीलतक दल है।
party of India.
 आस दल की स्थापना 1939 में हुइ थी।
 This party was established in 1939.
 नेताजी सुभाषचन्ि बोस नें आस दल की स्थापना की
 Netaji Subhash Chandra Bose had founded this
थी।
party.  आस दल के महासलचव देवव्रत लवश्वास हैं।
 The general secretary of this party is Devvrat
Biswas.

135. Answer. (c) ईत्तर. (c)


 Netaji renamed Andaman as Shaheed Dweep  नेताजी ने ऄंडमान का नया नाम शहीद िीप तथा
and Swaraj Island of Nicobar. लनकोबार का स्वराज्य िीप रखा गया।

 The flag of independent India was also hoisted  30 कदसम्बर 1943 को आन िीपों पर स्वतन्त्र भारत
का ध्वज भी फहरा कदया गया। आसके बाद नेताजी
on these islands on 30 December 1943. After this
सुभाष चन्ि बोस ने हसगापुर एवं रं गून में अज़ाद लहन्द
Netaji Subhash Chandra Bose made the णौज का मुख्यालय बनाया।
headquarters of Azad Hind Fauj in Singapore and
Rangoon.

136. Answer. (d) ईत्तर. (d)


Under the leadership of Sir Tej Bahadur Sapru in the  अजाद हहद बचाव सलमलत में कांग्रेस की लडफें स टीम
में सर तेज बहादुर सप्रू के नेतृत्व में मुल्क के ईस समय
defense team of Congress in Azad Hind Defence
के कइ नामी-लगरामी वकील भूलाभाइ देसाइ , सर
Committee, many well-known lawyers of the country like
कदलीपहसह, असफ ऄली , पं॰ जवाहरलाल नेहरू ,
Bhulabhai Desai, Sir Dilip Singh, Asaf Ali, Pandit
बख्शी सर टेकचंद , कै लाशनाथ काटजू , जुगलककशोर
Jawaharlal Nehru, Bakshi Sir Tekchand, Kailashnath
खन्ना, सुल्तान यार खान , राय बहादुर बिीदास ,
Katju, Jugalkishore Khanna , Sultan Yar Khan, Rai
पी.एस. सेन, रघुनंदन सरन अकद शालमल थे जो खुद
Bahadur Badridas, P.S. Sen, Raghunandan Saran etc.
आन सेनालनयों का मुकदमा लडने के ललए अगे अए
who themselves came forward to fight the case of these
थे।
fighters.  सर तेज बहादुर सप्रू की ऄस्वस्थता के कारण वकील
 Due to the ill health of Sir Tej Bahadur Sapru, भूलाभाइ देसाइ ने अजाद लहन्द फौज के तीनों वीरों
lawyer Bhulabhai Desai advocated on behalf of लसपालहयों की तरफ से वकालत की।

the three brave soldiers of the Azad Hind Fauj.

137. Answer. (a) ईत्तर. (a)

©MakeIAS www.makeias.in 9899282107, 8700476287


Wavell Plan वेवेल प्लान
 In October 1943, Lord Wavell replaced Linlithgow  ऄरटू बर,1943 को ललनललथगो की जगह लाडत वेवेल
as the Viceroy of India. The situation in India at भारत के वायसराय बन कर अये। आस समय भारत की
this time was very tense. लस्थलत ऄत्यलधक तनावपूणत थी।
 वेवेल लस्थलत को सामान्य बनाने की कदशा मे प्रयास
 Wavell, while trying to normalize the situation,
करते हुए सवतप्रथम भारत छोडो अंदोलन के समय
first released the members of the Congress लगरफ्तार कांग्रेस कायतसलमलत के सदस्यों को ररहा
Working Committee arrested during the Quit ककया।
India Movement.  14 जून,1945 को वेवेल योजना प्रस्तुत की गइ ,

 The Wavell plan was presented on June 14, लजसकी प्रमुख शतें आस प्रकार थी-
o के न्ि में एक नइ कायतकारी पररषद का गिन
1945, the main conditions of which were as
हो, लजसमें वायसराय तथा कमांडर आन चीफ
follows- के ऄलावा शेष सदस्य भारतीय हों।
o A new Executive Council should be o कायतकारी पररषद एक ऄंतररम व्यवस्था थी ,
formed at the Center, in which apart from लजसे तब तक देश का शासन चलाना था, जब
the Viceroy and the Commander-in-Chief, तक की एक नये स्थायी संलवधान पर अम
the rest of the members should be सहमलत नहीं हो जाती।

Indians.
o The Executive Council was an interim
arrangement, which was to govern the
country until a new permanent
constitution was agreed upon.

138.Answer. (c) ईत्तर. (c)


Cabinet mission plan कै लबनेट लमशन योजना
In January 1946, Prime Minister Attlee, leader of Britain's जनवरी, 1946 को लब्रटेन की लेबर पाटी के नेता प्रधानमंत्री
Labor Party, decided to send a Parliamentary Party एटली ने भारतीय नेताओं से ऄनौपचाररक स्तर पर वातात करने
के ललए एक संसदीय दल (कै लबनेट लमशन) को भारत भेजने का
(Cabinet Mission) to India to hold informal talks with
लनणतय ललया।
Indian leaders.
29 माचत, 1946 को कै लबनेट लमशन भारत अया। कै लबनेट
The Cabinet Mission came to India on 29 March 1946. लमशन के सदस्यों में शालमल थे-
The members of the Cabinet Mission included- 1. सर स्टेफडत कक्रप्स (व्यापार बोडत के ऄध्यक्ष)
1. Sir Stafford Cripps (Chairman of the Board of 2. श्री ए.वी.ऄलेरजेंडर (एडलमरै ललटी के प्रथम लाडत या
Trade) नौसेना मंत्री) तथा
3. पैलथक लारें स (भारत सलचव)
2. Shri AV Alexander (1st Lord of the Admiralty or
Minister of the Navy) and
3. Pethick Lawrence (India Secretary)

139.Answer. (b) ईत्तर. (b)


 The Ghadar Party was founded in 1913 in San  संयुक्त राज्य ऄमेररका के सैन फ़्ांलसस्कों में 1913 में
Francisco, United States. ‘गदर पाटी’ की स्थापना की गइ।

©MakeIAS www.makeias.in 9899282107, 8700476287


 Lala Hardayal was the main worker of this party.  लाला हरदयाल आस पाटी के प्रमुख कायतकतात थे। गदर
नामक पलत्रका का भी प्रकाशन हुअ लजसमें ऄंग्रेजों के
A magazine named Gadar was also published in
ऄत्याचार और शोषण का खुला लवरोध ककया जाता
which the atrocities and exploitation of the British
था।
were openly opposed.  गदर पाटी का ईद्देश्य स्थानीय क्रांलतकाररयों की
 The objective of the Ghadar Party was to revolt सहायता कर ऄंग्रेजों के लवरूद्ध लविोह करना था।
against the British by helping the local  गदर पाटी के संस्थापक ऄध्यक्ष सरदार सोहन हसह
भाकना थे।
revolutionaries.
 The founder president of the Ghadar Party was
Sardar Sohan Singh Bhakna.

140. Answer. (b) ईत्तर. (b)


 At the same time a Shore committee was formed  कामागाटामारू प्रकरण से ईत्पन्न लस्थलतयों के लखलाफ
in Canada under the leadership of Hussain ईसी समय कनाडा में हुसैन रहीम , बलवंत हसह ,
सोहन लाल पािक की ऄगुवाइ में एक शोर कमेटी
Rahim, Balwant Singh, Sohan Lal Pathak against
बनाइ गइ थी
the situations arising out of the Komagata Maru
episode.

141.Answer. (d) ईत्तर. (d)


 Toshimaru incident, 1914- This episode was  तोषामारू काण्ड, 1914. 1914द्ध यह प्रकरण गदर
related to the leaders of the Ghadar Party. पाटी के नेताओं से संबंलधत था।
 आस योजना के सूत्रधार खुदाइ सेना से जुडे मौलवी
 The architect of this plan was Maulvi Ubaidulla
ईबैदल्ु ला हसधी थे लजन्होंने कु छ गुप्त पत्र महमूद हसन
Sindhi associated with the Khudai army, who
को पीले लसल्क पर फारसी भाषा में ललखे , ककतु ये
wrote some secret letters to Mahmud Hasan in ऄंग्रेजों के हाथ लग गए और यह योजना ऄसफल हो
Persian language on yellow silk, but these fell in गयी। आलतहास में आस समूचे प्रकरण को लसल्क पेपर
the hands of the British and this plan failed. This षड्यंत्र के नाम से जाना जाता है।

entire episode in history is known as the Silk


Paper Conspiracy.

142. Answer. (d) ईत्तर. (d)


Rise of Militantism: ईग्रराष्ट्रवाद का ईदय:
 The old Congress leaders and their alms-begging  कांग्रेस के पुराने नेताओं और ईनकी लभक्षा-याचना की
नीलत ने कांग्रेस की लस्थलत हास्यास्पद बनाकर रख दी
policy had kept the Congress position ridiculous.
थी। आसललए कांग्रेस में एक नए तरूण दल का ईदय
Therefore, a new young party emerged in the हुअ जो पुराने नेताओं के ढोंग एवं अदशों का कडा
Congress, which emerged as a strong critic of the अलोचक बन कर ईभरा। फलतः कांग्रेस दो गुटों में
pretense and ideals of the old leaders. As a लवभालजत होने लगी यथा ईदारवादी और ईग्रवादी या
नरमपंथी और गरमपंथी।
result, the Congress began to split into two
 लोकमान्य लतलक , लबलपन चन्ि पाल , ऄरलवन्द घोष
factions, the liberals and the extremists or the
और लाला लाजपत राय गरमपंथी वगत के मुख्य
moderates and the extremists.
नेतृत्वकतात थे तथा आनका ईद्देश्य “पूणत स्वराज ” की
 Lokmanya Tilak, Bipin Chandra Pal, Arobindo

©MakeIAS www.makeias.in 9899282107, 8700476287


Ghosh and Lala Lajpat Rai were the main leaders प्रालप्त था।
of the extremist class and their aim was to
achieve "Purna Swaraj".

143.Answer. (c) ईत्तर. (c)


 Moderates - Gopal Krishna Gokhale, Ferozshah  नरमपंथी वगत - गोपाल कृ ष्ण गोखले , कणरोज़शाह
Mehta, Surendranath Banerjee, Dadabhai Naoroji मेहता, सुरेन्िनाथ बनजी, दादाभाइ नौरोजी

144. Answer. (c) ईत्तर. (c)


 Moderates - Constitutional Reforms, Increasing  नरमपंथी वगत - संवैधालनक सुधार, सरकारी सेवाओं में
Indian Participation in Government Services भारतीयों की सहभालगता बढाना
 गरमपंथी वगत - अवश्यकता पडने पर हहसक साधनों
 Extremist- Did not refrain from violent means
से भी परहेज नहीं
even when necessary

145. Answer. (c) ईत्तर. (c)


Reasons for the rise of militant nationalism (1905–1915) ईग्र राष्ट्रवाद के ईदय के कारण (1905-1915)
 Identifying the true character of British rule  लब्ररटश शासन के सही चररत्र की पहचान
 अत्म लवश्वास का संदेश
 Message of self confidence
 लशक्षा और बेरोजगारी में वृलद्ध
 Rise in education and unemployment  कांग्रेस की ईपललजधयों से ऄसंतोष
 Dissatisfaction with the achievements of  कजतन की प्रलतकक्रयावादी नीलतयााँ
Congress  समकालीन ऄन्ततराष्ट्रीय प्रभाव
 ईग्र लवचारों से प्रेररत नेताओं का प्रादुभातव
 Curzon's Reactionary Policies  ईग्रवादी दल के ईद्देश्य तथा कायतक्रम
 Contemporary International Impact
 The emergence of leaders driven by radical ideas
 Objectives and Programs of the Extremist Party

146. Answer. (b) ईत्तर. (b)


Rebellion Leader लविोह - नेतृत्वकतात
 Munda Rebellion - Birsa Munda  मुण्डा लविोह - लवरसा मुण्डा
भील लविोह - सेवरम
 Bhil Rebellion - Sevaram
 कू का लविोह - भगत जवाहरमल , बालक हसह एवं
 Kuka Rebellion - Bhagat JawaharMal, Balak
रामहसह
Singh and Ram Singh
 संथाल लविोह - लसद्दू और कान्हू
 Santhal Rebellion - Siddu and Kanhu

147. Answer. (c) ईत्तर. (c)


 The Simon Commission was appointed by the  साआमन कमीशन की लनयुलक्त लब्ररटश प्रधानमंत्री ने सर
British Prime Minister under the leadership of Sir जॉन साआमन के नेतृत्व में की थी।

John Simon.  आस कमीशन में सात सदस्य थे , जो सभी लब्रटेन की


संसद के मनोनीत सदस्य थे। यही कारण था कक आसे
 The commission consisted of seven members, all
'श्वेत कमीशन' कहा गया।
of whom were nominated members of the British

©MakeIAS www.makeias.in 9899282107, 8700476287


Parliament. This was the reason why it was
called 'White Commission'.

148.Answer. (d) ईत्तर. (d)


Withdrawal of Partition of Bengal बंगभंग की समालप्त
 On 12 December 1911, a court was held in Delhi,  सन् 1911 के 12 कदसम्बर को कदल्ली में एक दरबार

in which Emperor V George, Empress Mary and हुअ, लजसमें सम्राट् पंचम जाजत , सम्राज्ञी मेरी तथा

India Secretary Lord Crew had come. भारत सलचव लाडत क्रू अए थे।
 आस दरबार के ऄवसर पर एक राजकीय घोषणा-िारा
 On the occasion of this court, a state declaration
पलश्चम और पूवी बंगाल के बाँगला भाषी आलाकों को
was ordered to bring the Bengali speaking areas एक प्रांत में लाने का अदेश ककया गया।
of West and East Bengal into one province.

149. Answer. (a) ईत्तर. (a)


 Extremist class - They did not refrain from violent  गरमपंथी वगत - अवश्यकता पडने पर हहसक साधनों
means when needed, they had full loyalty to the से भी परहेज नहीं , आन्हें जनसामान्य की शलक्त एवं
त्याग की भावना में पूणत लनष्ठा थी।
power of the masses and the spirit of sacrifice.

150. Answer. (a) ईत्तर. (a)


Khilafat Movement लखलाफत अंदोलन
 भारतीय मुसलमानों ने लखलाफत अंदोलन अरं भ
 Indian Muslims started the Khilafat Movement.
ककया। मौलाना मोहम्मद ऄली और शौकत ऄली आस
Maulana Muhammad Ali and Shaukat Ali were अंदोलन के प्रभावशाली नेता थे।
the influential leaders of this movement.  गााँधीजी ने आसे लहन्दू-मुलस्लम एकता का स्वणत ऄवसर
 Gandhiji considered it a golden opportunity for समझा। ऄतः गााँधीजी ने आस अंदोलन का समथतन
ककया।
Hindu-Muslim unity. So Gandhiji supported this
 24 नवंबर, 1919 को कदल्ली में ऄलखल भारतीय
movement.
लखलाफत सम्मेलन हुअ , लजसमें गााँधीजी को ऄध्यक्ष
 On November 24, 1919, the All India Khilafat
चुना गया।
Conference was held in Delhi, in which Gandhiji
was elected president.

©MakeIAS www.makeias.in 9899282107, 8700476287

You might also like